Вы находитесь на странице: 1из 104

ISTQB Certified Tester Foundation Level

"I really liked the simple format of the book. There was no 'clutter' to take away from the actual purpose [of practicing for and passing the exam]. The reasons why a particular answer is more correct than another were very helpful."

-D. E. Ketch

1

Sample Exam Questions:

ISTQB Certified Tester foundation level

Second Edition

John A. Estrella, PhD, CTFl Maria C. Estrella, sse, CTFL

2

3

ISTQB, CTFL, and other terms may be registered trademarks or trademarks of their respective owners. The contents of this book were produced independently and have not been authorized, sponsored. or otherwise approved by the

International Software Testing Qualifications Board (ISTQB) or any of its national testing boards.

Order this book online at http://www.agilitek.com/producrs.html or email comacr@agHitek.com. Exa m simulation software based on this book is also available.

© Copyright 2007 John A. Estrella and Maria C. Estrella

All right reserved. No part of this book may be reproduced, stored in a retrieval system, or transrnitted, in any form or by any means, electron ic, mechanical, photocopying, recording, or otherwise, without the written prior permission of the author.

Agilitek Corporation 213-21 Main Street North Markham, ON L3P lX3 Canada

Phone: (905) 201-3085 Fax: (90S) 201-3086

You can also order this book online at www.traff.ord.com/07-1l49 or email orders@trafford.com

Most Trafford titles arc also available at major online book retailers.

Note for Librarians: A cataloguing record for this book is available from Library and Archives Canada at www.collectionscanada.ca/amicus/index-e.html

Printed in Vicroria, Be, Canada.

ISBN: 978-1-4251-3133-3

www.trafford.com

10 9 8 7 6 5 4 3 2

4

5

About the Authors

John A. Estrella, PhD, CMC, PMP, CBAP, CTFL, CSTE, SCP, MCP, is the President of Agilitek Corporation, a management consulting firm that specializes in information technology project management, business analysis, and software testing.

As a consultant, author, and speaker with extensive experience in Asia, North America, and Europe, John helps senior managers and executives in addressing issues associated with complex, large-scale, and risky projects. Some of his clients include Dell, SMa Financial Group, Learni!1g Tree International, and all levels of government.

Maria C. Estrella, SSe, CTFL, is the Director of Agilitek Corporation. She started her career with Procter & Gamble. While at Andersen Consulting (now Accenture), Maria worked on various technical consulting projects in Asia and North America. She worked as a senior systems analyst for ManuHfe Financial in Canada prior to joining AgiHtek. Maria holds a Bachelor of Science in Mathematics, with a major in Computer Science.

Contact Information

Agilitek Corporation

213-21 Main Street North Markham, Ontario L3P 1X3 Canada

Phone: (90S) 201-3085 Fax: (905) 201-3086

http://www.agilitek.com http://sampleexamquestions.com http://software-testing-information.com

6

7

Preface

Dedication

We wrote this book with four categories of readers in mind: planners, crammers, refreshers, and teachers.

To the Estrellas, Alfortes, and Buenviajes of General Luna, Quezon, Philippines-this book is dedicated to all of you!

Planners are very deliberate in their approach to taking the exam. They thoroughly research the topic and allocate sufficient time to prepare for the exam. These types of readers can quickly read "Chapter 1: Overview" and then dive right into Chapters 2, 4, and 6 to try the three sets of sample exam questions. If you feel very confident about taking the actual exam after answering the practice questions, you may skip the solutions in Chapters 3, 5, and 7.

-John A. Estrella

To lola Felicing, Lola Rosa, Tatay Norman, Nanay Zeny, and carlos-thank you very much for your inspiration and support!

-Maria (Chary) C. Estrella

Crammers should make sure to carefully read "Chapter 1:

Overview" before attempting the sample exam questions in Chapters 2, 4, and 6. If you did not score well on the practice questions, carefully read the solutions in Chapters 3[ 5, and 7.

If you previously failed the exam or have taken an exam preparation course but would like to further boost your confidence, we consider you to be part of the refreshers group. Try the sample exam questions to determine your areas of strength and weakness. Read the corresponding sections in the syllabus for your weak areas and focus on the same topics in the solution chapters.

Teachers may use the book as a valuable course supplement. They can measure students' understanding of each ISTQB Certified Tester Foundation Level Syllabus chapter by focusing on spedtlc questions from that particular chapter.

8

9

Disclaimer

Acknowledgment

Although every effort was made to ensure the accuracy of the contents of this book, we cannot guarantee 100% correctness of the information contained herein, Likewise, we cannot guarantee with 100% certainty that reading this book will result in you passing the exam,

This book would not have been possible without the dedicated effort of numerous individuals, including the founders and supporters of the International Software Testing Qualifications Board (ISTQ8). The authors acknowledge that this book relies heavily on the ISTQB Certified Tester Foundation Syllabus and Standard Glossary of Terms Used in Software Testing. In some cases, certain phrases were used verbatim to ensure strict adherence to the syllabus and the glossary. Please accept our sincerest gratitude for your hard work!

We recoqnize that you enjoy swatting software bugs and may thrive on finding factual anomalies, spelling mistakes, and grammatical errors. If you find any of these in the book, kindly send your discoveries, comments, and suggestions to contact@agiJitek.com.

Good luck on the exam!

We also would like to thank Luisito Pangilinan, A. J. Sobczak, Brian Yao, Juran Liu, Darlene E. Ketch, Ashna Datye, Debra Fernandes, and Melvin Tancio for their assistance with this book.

10

11

Table of Contents

Chapter: 1: Overview .•....•....................... " ..............•......•....... 15

ISTQB " 15

Syllabus and Exam , , 16

Question Format 18

Basic , " , 18

Roman (Simple) ., 19

Roman (Complex) , " , 19

Question Breakdown , , , , 20

Checklist ' , , 21

Chapter 2: Practice Questions A " .. , ,23

Chapter 3: Practice Solutions A 37

Cha pter 4: Practice Questions B " 83

Chapter 5: Practice Solutions B 97

Chapter 6: Practice Questions C 141

Chapter 7: Practice Solutions C 155

References " , 197

Index ,., , ...•.. , 199

12

13

This book was designed to help you prepare, in a time-efflcient manner, for the International Software Testing Qualification Board (ISTQB) Certified Tester Foundation Level exam. The distilled format of the book will provide you with the fundamental knowledge in software testing as an aid to passing the exam with the minimum amount of effort.

To ensure accurate adherence to the exam requirements, this book strictly followed the Certified Tester Foundation Level Syllabus. Relevant terminologies were derived from the Standard Glossary of Terms Used in Software Testing. Practice questions were written based on the ISTQB Foundation Exam Format and Question Writing Guidelines. The IEEE Standard for Software Test Documentation (IEEE Std 829-1998) provides details that will help you answer questions related to documentation.

All of the aforementioned documents are available on the Internet. If you cannot locate these documents, subscribe to our newsletter by sending an e-mail tosubscribe@agilitek.com. We will provide you with additional tips to help pass the exam. Do not hesitate to contact us for corrections, questions, and suggestions via Agilitek's website at http://www.agilitek.com.

ISTQB

ISTQB certifications are administered by their respective national testing boards in each member country. For example, the Canadian Software Testing Board (CSTB) manages ISTQB exams in Canada. Likewise, the American Software Testing Quality Board (ASTQB) coordinates ISTQB exams in the United States of America. In the United Kingdom, the British Computer Society's (BCS) Information Systems Examination Board (ISEB) Foundation Certificate in Software Testing leads to the ISTQB Certified Tester Foundation Level.

14

15

Sample Exam Questions: ISTQB Certified Tester Foundation Level

Sample Exam Questions: rSTQB Certified Tester Foundation Level

Syllabus and Exam

K1 K2 K3 Total
Ch 1 4 3 0 7
Ch 2 4 2 0 6
Ch 3 2 1 0 3
Ch 4 4 2 6 12
Ch 5 3 3 2 8
Ch 6 3 1 0 4
Total 20 12 8 40 The syllabus categorizes learning objectives into three cognitive levels.

• K1: remember, recognize, and recall

.. K2: understand, explain, give reasons, compare, classify, and summarize

• K3: apply

You will have one hour to answer 40 multiple-choice questions. There are four choices per question (response options), including one and only one correct answer (key option), Roughly half (50%) of the questions will be at the K11evel (20 questions), 30% at the K2 level (12 questions), and 20% at the K3 level (8 questions),

The list below shows the approximate breakdown of exam questions per chapter. When you take the exam, keep in mind that the breakdown may not be exactly as shown here and that the ISTQB may adjust it as appropriate. There are very slight variations in the passing score per country. Some countries require a passing score of 60% while others require you to correctly answer 25 out of 40 questions (62.5%),

• Chapter 1- 18% (7 questions)

.. Chapter 2 r: 16% (6 questions)

.. Chapter 3 - 7% (3 questions)

.. Chapter 4 - 29% (12 questions)

• Chapter 5 - 21% (8 questions)

• Chapter 6 - 9% (4 questions)

16

17

Sample Exam Questions: ISTQB Certified Tester Foundation Level

Sample Exam Questions: ISTQB Certified Tester Foundation Level

Question Format

Roman (Simple)

ISTQB uses two question formats: Basic and Roman. Basic format questions include a simple stem--the actual questionfollowed by four response options. In contrast, Roman format questions include several statements in the stem. The four response optlons-choices for potential answers-are based on combinations of the statements. The following examples demonstrate the different question formats used in the exam.

2. WhiCh factors contribute to humans making mistakes that can lead to faulty software?

1. Setting an aggressive schedule II. Integrating complex systems III. Allocating adequate resources IV. Failing to control changes

Basic

A. I and II are true; III and IV are false

B. II and IV are true; I and III are false

C. I, II, and IV are true; III is false

D. I, II, and III are true; IV is false

1. Which of the problems below BEST characterizes a result of software failure?

A. Damaged reputation

B. lack of methodology

C. Inadequate training

D. Regulatory compliance

As shown in sample question 2, Roman format questions have four statements labeled I, II, III, and IV in the stem. In some cases, the statements are labeled using small numerals (i, ii, iii, and iv), the alphabet (e.g., S, T, U, and V), or some other similar variations. The stem might contain three, four, or five statements. Keep In mind that the statements can be all true, all false, or a combination of true and false. The correct answer must fully and accurately describe the truthfulness of the statements.

In sample question 1, the question stem is "Which of the problems below BEST characterizes a result of software failure?" If it is necessary to avoid ambiguity or to highlight the main thrust of the question, sometimes a word will be emphasized in the stem, such as "BEST" in this example. Pay close attention to the emphasized word (it can be in capital letters or in bold format), or you may misunderstand the main point of the que;:tion. Other examples of emphasis words include key, least, mam, major, more, most likely, most, must, often, optionally, and usually.

Roman (Complex)

The correct answer is called the key option, Incorrect or inferior choices are called distracters. Within the context of a specific question, a distracter may be "incorrect" simply because it is inferior to the key option. In other cases, a distracter may also be a true statement but an incorrect response option if associated with the current question.

Sample question 3 shows a more difficult variation of the Roman type of question, Although the question may still test for the same level of knowledge (usually K2 or K3), there are two sets of statements in the stem. Similar to the other types of questions, there will also be four choices. However, the first set of statements (W, X, Y, and Z) needs to be properly matched to the second set of statements (1, 2, 3, and 4) in order to arrive at the correct answer.

18

19

Sample Exam Questions: ISTQB Certified Tester Foundation Level

Sample Exam Questions: ISTQB Certified Tester Foundation Level

3. Which tests are BEST described by the following characteristics?

Checklist

W. Component testing

X. Integration testing

Y. Alpha testing

Z. Robustness testing

-;-=-

o Read the Certified Tester Foundation Level Syllabus (ISTQB, 2007a). You can read the entire syllabus and then try to answer all of the 40 practice questions in one sitting. To simulate the actual exam, find a quiet room to avoid distractions and allocate 60 minutes to answer all of the questions. Alternatively, you can read the syllabus chapter by chapter and then try to answer just the respective questions.

1. Testing the interaction between components 2; Fixing defects as soon as they are found

3. Automating test cases before coding

4. Testing separately testable components

o Syllabus Chapter 1 - Questions 1 to 7

o Syllabus Chapter 2 - Questions 8 to 13

o Syllabus Chapter 3 - Questions 14 to 16

o Syllabus Chapter 4 - Questions 17 to 28

o Syllabus Chapter 5 - Questions 29 to 36

o Syllabus Chapter 6 - Questions 37 to 40

A. W1, X4, Y3, and Z2

B. W2, W4, xi. and Z3

C. W2, W3, W4, and Xl

D. W3, Xl, X2, and X4

Question Breakdown

lJ Mark the relevant terms in the Standard Glossary of Terms Used in Software Testing (ISTQB, 2007b), As you read the syllabus, keep an eye out for sections titled "Terms" in each chapter. Highlight the appropriate word in the glossary.

At the chapter level, the breakdown of exam questions based on cognitive levels is more subjective than prescriptive. As you prepare for the exam, take note of Kl, K2, and K3 notations next to each major heading in the syllabus. A cognitive level of K1 indicates that you will be tested to "remember, recognize, and recall" the material using the basic multiple-choice question format. K2 and K3 types of questions will most likely use the Roman type Qf questions. K2 attempts to check if you can "understand, explain, give reasons, compare, classify, and summarize" various concepts as described in each of the question stem statements. In contrast, K3 requires that you "apply" a concept; which in general will be more difficult than the tasks in K1 and K2 questions. K3 questions may also include simple diagrams or short programming codes as part of the stem.

o Examine the ISTQB Foundation Exam Format and Question Writing Guidelines (ISTQB, 2005).

o Familiarize yourself with the IEEE Standard for Software Test Documentation (IEEE 5td 829-1998).

Good luck on the exam!

20

21

Sample Exam Questions: ISTQB Certified Tester Foundation level

1. Which of the problems below BEST characterize a result of software failure?

A. Damaged reputation

B. Lack of methodology

C. Inadequate training

D. Regulatory compliance

2. What should be taken into account to determine when to stop testing?

I. Technical risk II. Business risk

III. Project constraints

IV. Product documentation

A. I and II are true; III and IV are false 8. III is true; If II, and IV are false

C. I, II, and IV are true; III is false

D. I, II and III are true; IV is false

3. What is the process of analyzing and removing causes of failures in software?

A. Validation

B. Testing

C. Debugging

D. Verification

22

23

Sample Exam Questions: ISTQB Certified Tester Foundation Level

Sample Exam Questions: ISTQB Certified Tester Foundation Level

4. Which general testing principles are characterized by the descriptions below?

7. What principle is BEST described when test designs are written by a third-party?

W. Early testing

X. Defect clustering

Y. Pesticide paradox

Z. Absence-of-errors fallacy

A. Exploratory testing

B. Independent testing

C. Integration testing

D. Interoperablllty testing

1. Testing should start at the beginningof the project

2. Conformance to requirements and fitness for use

3. Small number of modules contain the most defects

4. Test cases must be regularly reviewed and revised

8. Which test levels are USUALLY included in the common type of V-model?

A. W1, X2, Y3 and Z4

B. W1, X3, Y4 and Z2

C. W2, X3, Y1 and Z4

D. W1, X4, Y2 and Z3

A. Integration testing, system testing, acceptance testing and regression testing

B. Component testing, integration testing, system testing and acceptance testing

C. Incremental testing, exhaustive testing, exploratory testing and data driven testing

D. Alpha testing, beta testing, black-box testing and white-box testing

5. Which of the following MAIN activity is part of the fundamental test process?

A. Initiating and planning

B. Documenting root-causes

C. capturing lessons learned D .. Planning and control

9. What test can be conducted for ofHhe-shelf software to get market feedback?

6. Which of the following are MAJOR test implementation and execution tasks?

A. Beta testing

B. Usability testing

C. Alpha testing

D. COTS testing

I. Repeating test activities II. Creating test suites

III, Reporting discrepancies IV. Logging the outcome

V. Analyzing lessons learned

10. Who OFTEN performs system testing and acceptance testing respectively?

A. Senior programmers and professional testers

B. Technical system testers and potential customers

C. Independent test team and users of the system

D. Development team and customers of the system

A. II, III and IV

B. I, III, IV and V

C. I, II, III and IV

D. III, IV and V

24

25

Sample EXam Questions: ISTQB Certified Tester Foundation Level

Sample Exam Questions: ISTQB Certified Tester Foundation Level

11. What is the key difference between (a) contract and regulation acceptance testing, and (b) aJpha and beta testing?

15. Ina formal review, who is primarily responsible for the documents to be reviewed?

A. (a) are performed outside the company and (b) are conducted by the test team

B. (a) are conducted by regulators and (b) are performed by system administrators

C. (a) are mandatory test for government applications and (b)

are usually optional .

D. (a) are for custom-developed software and (b) are for offthe-shelf software

A. Author

B. Manager

C. Moderator

D. Reviewers

16. Who typically use static analysis tools?

12. Which test measures the system at or beyond the limits of its specified requirements?

A. Customers and users

B. Developers and designers

C. Business and systems analysts

D. System and acceptance testers

17. Which aspects of testing will establishing traceability help?

A. Structural testing

B. Stress testing

C. Error guessing

D. Black-box testing

13. Which test ensures that modifications did not introduce new problems?

A. Configuration management and test data generation

B. Test case speCification and change control

C. Test condition and test procedure specification

D. Impact analysis and requirements coverage

18. Features to be tested, approach, item pass/fail criteria and test deliverables should be specified in which document?

A. Stress testing

B. Black-box testing

C. Structural testing

D. Regression testing

14. Which typical defects are easier to find using static instead of dynamic testing?

A. Test case specification

B. Test procedure specification

C. Test plan

D. Test design specitlcation

L. Deviation from standards

M. Requirements defects

N. Insutflcient maintainability

O. Incorrect interface spedflcations

19. Which test technique is based on requirements specifications?

A. t, Mr Nand 0

B. Land N

C. L, Nand 0

D. l, M and N

A. White-box technique

B. Component testing

C. Black-box technique

D. Data driven testing

26

27

Sample Exam Questions: ISTQB Certified Tester Foundation Level

Sample Exam Questions: ISTQB Certified Tester Foundation Level

20. Which test design techniques should a tester use to respectively achieve the following: (a) check the documented features of the system, (b) ensure 100% decision coverage, and (c) detect likely defects and distribution?

23. Which test suite will check for an invalid transition using the diagram below?

A. Specification·based, data driven testing, and defect density

techniques .

B. Specification-ba'sed, branch coverage, and exploratory

techniques ..

c. Strudure·based, equivalence partitioning, and exploratory techniques

D. Specification-based, structure-based I and experience-based techniques

r> \::J

21. What technique captures system requirements that contain logical conditions?

A. Boundary value

B, Equivalence partition C, Decision table

D, State transition

A. 50 - 51- 52 - 53 - 51- 54

B. 50 - Sl - S4 - 51 - S2 - 53

C. 50 - 51 - 53 - 51 - 52 - Sl

D. 50 - 51 - 52 - 53 - Sl - 52

22. Input and output combinations that will be treated the same way by the system can be tested using which technique?

24. How are integration testing and use case testing similar and dissimilar?

A. Both checksfor interactions: integration for components, use case for actors

B. Both are black-box techniques: integration is low-level, use case is high-level

C. Both are static testing: developers perform integration, users execute use case tests

D. Both are V&V techniques: integration is for validation, use case is for verification

A. Boundary value

B, Equivalence partition

C. Decision table

D. State transition

28

29

Sample Exam Questions: ISTQB Certified Tester Foundation Level

Sample Exam Questions: ISTQB Certified Tester Foundation Level

25. How many test cases are needed to achieve 100% decision coverage?

28. Which input combinations will a knowledgeable tester MOST UKE;L Y use to uncover potential errors when testing a surname field?

if (p == q) {
s = s + 1·
,
if (s < 5) {
t == IQi
} ~"
} else if (p > q) {
t = 5;
r
A. 3
B. 6
C. 5
D. 4 A. Johnson, de la Cruz and Morgan

6. Go, Stephanopoulous and Venkatsewaran

C. Smit, Smyth and Smithsonian

D. O'Brien, Zeta-Jones and Young Pow

29. Which of the following demonstrates independence in testing?

26. What analysis determines which parts of the software have been executed?

J. Independent testers are external to the organization

K, Independent testers are part of the development team L. Independent testers are from the user community

M, Programmers who wrote the code serve as independent testers

N. Customers who wrote the requirements serve as independent testers

A. Impact analysis

B. Code coverage

C. Gap analysis

D. CycJomatic complexity

27. Based on ~he e~ror guessing test design technique, which of the followmg will an experienced tester MOST LIKELY test, in calendar software?

A. J, Land N

B. J, K, Land N

C. K, M and N D, J, L, M and N

30. Which of the following is a KEY task of a tester?

i.

A. Reviewing tests developed by others

B. Writing a test strategy for the project

C. Deciding what should be automated

D. Writing test summary reports

First two letters of the month, e.g., MA can represent March or May

ii. First letter of the day, e.g., T can mean Tuesday or

Thursday iii. Leap year

iv. Number of days in a month v. Three-digit days and months

31. In software testing, what is the MAIN purpose of exit criteria?

A. if ii, lv and v

B. iii and iv

C. if ii, iii and iv

D. i, H and v

A. To enhance the security of the system

B. To prevent endless loops in codes

C. To serve as an alternative or "Plan Btl

D. To define when to stop testing

30

31

Sample Exam Questions; ISTQB Certified Tester Foundation Level

Sample Exam Questions: ISTQB Certified Tester Foundation Level

32, Which test approaches or strategies are characterized by the descriptions below?

35, Based on the IEEE Standard for Software Test oocumentatoa (IEEE Std 829-1998), Which sections of the test incident report should the following details be recorded?

S, Analytical approaches

T, Model-based approaches

U. Methodical approaches

V. Consultative approaches

Sections

a) Test incident report identifier

b) Summary

c) Incident description

d) Impact

1. Relies on guidelines from domain experts

2. Includes error guessing and fault-attacks

3. Uses statistical information about failure rates

4. Focuses on areas of greatest risk

Details

A. 541 T31 U2, VI

B. 51, T2, U3, V4

C. 52, T3, Ul, V4

D. 53, T4, U2, V1

1. Unique identifier

2. Version level of the test items

3. Inputs

4, Expected results 5, Actual results

6. Anomalies

7. Date and time

33. Which of the following can be used to measure progress against the exit criteria?

W. Number of test cases that passed or failed

X. Number of defects found in a unit of code

Y. Dates for milestones and deliverables

Z. Subjective confidence of testers in the product

A. a: 1; b: 2 and 7; c: 3, 4 and 5; d: 6

B. a: 1; b: 6 and 7; c: 3,4 and 5; d: 7

C. a: 1; b: 2; c: 3, 4, 51 6 and 7

D. a: 1; b: 6 and 7; c: 3, 4 and 5

A. W, X, Y and Z

B. W, X and Y

C. Wand X

D. W, X and Z

36. Based on the IEEE Standard for Software Test Documentation (IEEE Std 829-1998), which of the following sections are part of the test summary report?

34. What type of risk includes potential failure areas in the software?

a) Test summary and report identifier

b) Summary

c) Variances

d) Anomalies

e) Comprehensive assessment

f) Approvals

A. Project risks

B. Product risks

C. Economic risks

D. Requirements risks

A. a, b, e and f

B. a, b, c, d and f

C. a, b, c, e and f

D. a, b, c and f

32

33

Sample Exam Questions: ISTQB Certified Tester Foundation Level

Sample Exam Questions: ISTQB Certified Tester Foundation Level

37, What is the name of a skeletal implementation of a software component that is used for testing?

Correct Answers and Cognitive Levels

A. Use case

B. Domain

C. Driver D, Stub

1 A K1
2 D K2
3 C K1
4 8 K2
5 D K1
6 C K2
7 B K1
8 8 K1
9 A K1
10 C K2
11 D K2
12 B K1
13 D K1
14 A K2
15 A K1
16 8 K1
17 D K2
18 C K3
19 C K1
20 D K3 21 C K1
22 8 K1
23 C K3
24 A K2
25 0 K3
26 B K1
27 C K3
28 D K3
29 B K3
30 A K1
31 D Kl
32 A K2
33 A K2
34 B Kl
35 C K3
36 C K2
37 D Kl
38 B K2
39 A Kl
40 A K1 ·38. Which of the following are potential benefits of using test support tools?

A. Ensuring greater consistency and minimizing software project risks

B. Reducing repetitive work and gaining easy access to test information

C. Performing objective assessment and reducing the need for training

D. Allowing for greater reliance on the tool to automate the test process

39. Which test support tool can be used to enforce coding standards?

A. Static analysis tool

B. Performance testing tool

C. Test comparator

D. Test management tool

40. What should be considered when introducinq a tool into an orga n ization?

A. Assessing the organizational maturity

B. Counting the number of systems to be tested

C. Calculating the ratio between programmers and testers

D. Reviewing the exit criteria of previous projects

34

35

Sample Exam Questions: ISTQB Certified Tester Foundation Level

1. Which of the problems below BEST characterizes a result of software failure? (Kl)

A. Damaged reputation B, Lack of methodology

C. Inadequate training

0, Regulatory compliance

Solution

This question is looking for a result of software failure, Lack of methodology and inadequate training are contributors to software failure-not results of software failure, Regulatory compliance can be a consequence of non-faulty software, Therefore, only A is the correct answer,

Answer A is correct.

Reference

ISTQB (2007a), p, 11.

36

37

Sample Exam Questions: ISTQB Certified Tester Foundation Level

I. Technical risk II. Business risk

III. Project constraints

IV. Product documentation

Sample Exam Questions: ISTQB Certified Tester Foundation Level

3. What is the process of analyzing and removing causes of failures in software? (Kl)

2. What should be taken into account to determine when to stop testing? (K2)

A. Validation

B. Testing

C. Debugging

D. Verification

A. I and II are true; III and IV are false

B. III is true; If II, and IV are false

C. II II, and IV are true; III is false

D. I, II, and III are true; IV is false

Solution

Solution

Validation confirms that the software is fit for use. In contrast, verification ensures that the software conforms to requirements. Testing shows failures caused by defects. Only debugging deals with finding the cause of the defect, fixing the code, and ensuring correct defect resolution. Validation, verification, and testing all involve some type of analysis but not the actual removal of failures.

Technical and business risks are important factors when deciding to stop testing. Likewise, project constraints (scope, time, and cost) directly influence the stopping decision. Product documentation is somewhat vague and out of context relative to the question.

Answer C is correct.

References

Answer D is considered correct.

ISTQB (2007a), p. 13 and ISTQB (2007b), pp, 13, 34 & 35.

Reference

ISTQB (2007a)1 p. 12.

38

39

Sample Exam Questions: ISTQB Certified Tester Foundation Level

. Sample Exam Questions: ISTQB Certified Tester Foundation Level

4. Which general testing principles are characterized by the descriptions below? (K2)

Reference

ISTQB (2007a), p. 14.

W. Early testing

X. Defect clustering

Y. Pesticide paradox

Z. Absence-of-errors fallacy

1. Testing should start at the beginning of the project

2. Conformance to requirements and 'fitness for use

3. Small number of modules contain the most defects

4. Test cases must be regularly reviewed and revised

A. W1, X2, Y3, and Z4

B. Wi, X3, Y4, and Z2

C. W2, X3, Yi, and l4

D. W1, X4, Y2, and Z3

Solution

As the principle implies, early testing (W) involves starting testing at the start of the project (1) by reviewing requirements, checking the design, and performing code walkthroughs-not toward the end of the software development life cycle when defects are more difficult and expensive to fix.

Defect clustering (X) indicates how most of the defects (3) can be attributed to very small units or modules of code.

The pesticide paradox (Y) states that running the same tests will eventually reach a point where no new defects will be found. Given this, test cases must be regularly reviewed and revised so that new defects can be uncovered (4).

lastly, the absence-of-errors fallacy (Z) highlights the importance of ensuring that the software is fit for its intended use-even after determining that the software conformed to the requirements (2).

Answer B is correct.

40

41

Sample Exam Questions: ISTQB Certified Tester Foundation Level

Sample Exam Questions: ISTQB Certified Tester Foundation Level

6. Which of the following are MAJOR test implementation and execution tasks? (K2)

5. Which of the following MAIN activities is part of the fundamental test process? (Kl)

A. Initiating and planning

B. Documenting root causes

C. Capturing lessons learned

D. Planning and control

.P

I. Repeating test activities II. Creating test suites

Ill. Reporting discrepancies IV. Logging the outcome

V. Analyzing lessons learned

Solution



planning and control,

analysis and design,

implementation and execution, evaluating exit criteria and reporting, and test closure activities .

A. II, III, and IV

B. I, III, IV, and V

C. I, II, III, and IV

D. III, IV, and V

The fundamental test process activities are:





Solution

Test implementation and execution activities transform test conditions into test cases-including setting up the test environment. V is part of test closure activities which explains why Band 0 are incorrect. Key option C is superior to A because it includes an additional correct task.

..

Documenting root causes and capturing lessons learned are detailed activities (not MAIN activities) and therefore Band C can be eliminated as correct options. Options A and D both incfude planning, which is a fundamental test process. After planning, the test process requires ongoing tracking and reporting. Because initiating is a one-time event, 0 is a superior choice to A.

Answer C is considered correct.

Reference

AnSwer D is considered correct.

lSTQB (2007a), p. 16.

References

ISTQB (2007a), p. 15 and logic,

42

43

Sample Exam Questions: ISTQB Certified Tester Foundation Level

Sample Exam Questions: ISTQB Certified Tester Foundation Level

7. What principle is BEST described when test designs are written by a third party? (Kl)

8. Which test levels are USUALLY included in the common type of V-model? (Kl)

A. Exploratory testing

B. Independent testing

C. Integration testing

D. Interoperability testing/

Solution

A. Integration testing, system testing, acceptance testing, and regression testing

B. Component testing, integration testing, system testing, and acceptance testing

C. Incremental testing, exhaustive testing, exploratory testing, and data driven testing

D. Alpha testing, beta testing, black-box testing, and white-box testing

Exploratory testing uses insights gained while testing to create new and improved test cases.

Solution

Independent testing ensures that someone other than the author of software tests it for defects so that an independent party can find problems overlooked by the author.

Integration testing exposes defects in the interfaces and interactions between components.

The left-hand side of the V-model, from top to bottom, generally includes defining the business need, specifying requirements, designing the system, and building the system. The right-hand side, from bottom to top, comprises four common test levels: component (unit) testing, integration testing, systems testing, and acceptance testing. These are the test levels included in key option B.

Interoperability testing checks the capability of the system when it interacts with other components or systems.

ISTQB (2007a), p. 17 and ISTQB (2007b), pp. 16 & 19,

With respect to the V&V concept (verification and validation), the left-hand side of the V-model is often associated with the verification process or static testing (conformance to requirements), and it involves walkthroughs, reviews, and inspections. The right-hand side of the V-model generally pertains to the validation process or dynamic testing (fitness for use).

Answer B is correct.

References

44

45

Sample Exam Questions: I5TQB Certified Tester Foundation level

Sample Exam Questions; ISTQB Certified Tester Foundation level

2. Prl)pars acceptance test cases

4. Pr8pa1t> system lest cases

9. What test can be conducted for off-the-shelf software to get

market feedback? (Kl)

A. Beta testing

B. Usability testing

C. Alpha testing

D. COTS testing

- ..... _- ............ ----- ......... _ - -- -- .... - -- ..... "' ... _- -- .......... --- --- - -- - ......

Solution

A thorough understanding of the V-rnodel will help you answer several questions in the exam, Given that there are four choices per question, you have a 25% chance of answering a question correctly through a random guess. If you go back to question 3, you can eliminate the verification and validation choices based on your knowledge of the V-model, Using the process of elimination, two choices would have been eliminated, thus, increasing the likelihood of answering the question correctly from 25% to 50%,

Usability testing determines how users will learn and use the system. Technically, there is "really" no such test called COTS testing. Beta testing serves as an external test for off-the-shelf software. Although alpha testing also can be used for off-theshelf software, it is generally implemented for internal acceptance purposes. Even though beta and usability testing are potential answers, beta testing is the superior choice by

definition. .

Answer A is considered correct.

References

ISTQB (2007a), p. 22 and ISTQB (2007b), pp. 7, 8 & 35.

Answer B is correct.

References

ISTQB (2007a), p. 20 and experience.

46

47

Sample Exam Questions: ISTQB Certified Tester Foundation Level

Sample Exam Questions: ISTQB Certified Tester Foundation Level

10. Who OFfEN performs system testing and acceptance testing, respectively? (K2)

11. What is the key difference between (a) contract and regulation acceptance testing and (b) alpha and beta testing? (K2)

A. Senior programmers and professional testers

B. Technical system testers and potential customers

C. Independent test team and users of the system

D. Development team and .custorners of the system

Solution

A. (a) are performed outside the company and (b) are conducted by the test team

B. (a) are conducted by regulators and (b) are performed by system administrators

C. (a) are mandatory test for government applications and (b) are usually optional

D. (a) are for custom-developed software and (b) are for off-the-shelf software

Although this question looks like a Kl type of question, it is really a K2 type of question because it requires you to know two distinct responsibilities: system testing and acceptance testing. Independent test teams conduct system testing, and customers/users of the system perform acceptance testing.

Solution

Reference

This is another example of a difficult K2 type of question. You need to know four concepts: two types of acceptance testing (contract and regulation) and two types of off-the-shelf testing (alpha and beta).

Answer C is correct.

ISTQB (2007a), p. 23.

A careful review of the response options will help eliminate A, B, and C. The term "test team" makes choice A incorrect because both alpha and beta testing are performed by potential customers. Likewise, "system administrators" make choice B incorrect. The first portion (a) of choice C is incorrect because contract and regulation acceptance testing can apply to all types of applications. Although incomplete, option D (a) is correct for contract acceptance testing and option D (b) is correct for both alpha and beta testing.

Answer D is considered correct.

References

ISTQB (2007a), p. 24 and logic.

48

49

Sample EXam Questions: ISTQB Certified Tester Foundation Level

Sample Exam Questions: ISTQB Certified Tester Foundation Level

13. Which test ensures that modifications did not introduce new problems? (Kl)

12. Which test measures the system at or beyond the limits of

its specified requirements? (Kl)

A. Structural testing

B. Stress testing

C. Error guessing

D. Black-box testing

A. Stress testing

B. Black-box testing

C. Structural testing

D. Regression testing

Solution

Solution

Structural testing is synonymous with white-box testing and relies on the analysis of the internal structure of a component.

Stress testing evaluates the system at or beyond the limits of its specified requirements.

Stress testing evaluates the system at or beyond the limits of its specified requirements,

Black-box testing derives test cases based on analysis of the specifications.

Error guessing relies on the tester's experience to anticipate potential defects and would therefore involve designing tests" to specifically expose those defects.

Structural testing is synonymous with white-box testing and relies on the analysis of the internal structure of a component.

Black-box testing derives test cases based on analysis of the specifications.

Regression testing ensures that no new problems were introduced in the unchanged portion of the software.

Answer 0 is correct.

Answer B is correct.

References

References

ISTQB (2007a), pp. 25-26 and ISTQB (2007b), pp. 8, 25, & 29.

ISTQB (2007a), p. 25 and ISTQB (2007b), pp. 8, 16, & 29.

50

51

Sample Exam Questions; ISTQB Certified Tester Foundation Level

14. Which typical defects are easier to find using static instead of dynamic testing? (K2)

L. Deviation from standards

M. Requirements defects

N. Insufficient maintainability

O. Incorrect interface spedflcations

A. L, M, N, and 0

B. Land N

C. L, Nt and 0

D. L, M, and N

Solution

This question can be answered using the concepts from the Vmodel, specifically, the left-hand side of the V (static testing / verification I conformance to requirements). There is no need for the actual execution of the code (dynamic testing / validation I fitness for use). AU of the statements L to 0 deal with defect prevention-artifacts that can be checked even before the first line of code is written.

Answer A is correct.

Reference

ISTQB (2007a), p. 29.

52

Sample Exam Questions: ISTQB Certified Tester Foundation Level

15. In a formal review, who is primarily responsible for the documents to be reviewed? (Kl)

A. Author

B. Manager

C. Moderator

D. Reviewers

Solution

There are five roles and responsibilities in a formal review:

• manager-plans the review,

• moderator-leads the review,

• author-owns the artifacts to be reviewed,

• reviewer-performs the actual review, and

• 5cribe-documents the results.

Answer A is correct.

Reference

ISTQB (2007a), p, 30.

53

Sample Exam Questions: ISTQB Certified Tester Foundation Level

16. Who typically use static analysis tools? (K1)

A. Customers and users

B. Developers and designers

C. Business and systems analysts

D. System and acceptance testers

Solution

Take note that the question asks for static analysis tools. Do not confuse the term with static testing. Static analysis tools are generally used for checking the codes against standards during development and for modeling the software during design. Customers, users, and business analysts have nothing to do with programming codes. System and acceptance testers deal with the higher level of the right-hand side of the V-model. Thus, answers A, C1 and 0 can be eliminated as valid choices.

Answer B is correct.

Reference

ISTQB (2007a), p. 33.

54

Sample Exam Questions: ISTQB Certified Tester Foundation Level

17. Which aspects of testing will establishing traceability help? (K2)

A. Configuration management and test data generation

B. Test case specification and change control

C. Test condition and test procedure spedficatlon

D. Impact analysis and requirements coverage

Solution

This question is an innocent looking K2 type of question. In order to arrive at the correct answer, both concepts in each enoree (notice the word AND) must be correct. Furthermore, all choices are plausible answers. However, option D is more encompassinq, which makes it superior to the other three choices,

Answer D is considered correct.

References

ISTQB (2007a), p. 36; ISTQB (2007b)1 p. 35; and logiC.

55

Sample Exam Questions: ISTQB Certified Tester Foundation Level

18. Features to be tested, approach, item pass/fail criteria, and test deliverables should be specified in which document? (K3)

A. Test case specification

B. Test procedure specification

C. Test plan

D. Test design specification

Solution

This K3 type of question requires not only familiarization with but also application of IEEE Standard for Software Test Documentation (IEEE Std 829-1998). Sections of a test plan include:

a) Test plan identifier

b) Introduction

c) Test items

d) Features to be tested

e) Features not to be tested

f) Approach

g) Item pass/fail criteria

h) Suspension criteria and resumption requirements

i) Test deliverables

j) Testing tasks

k) Environmental needs I) Responsibilities

m) Staffing and training needs n) Schedule

0) Risks and contingencies p) Approvals

Answer C is correct.

References

ISTQB (2007a), p, 36 and IEEE (1998), p. 3.

56

Sample Exam Questions: ISTQB Certified Tester Foundation level

19. Which test technique is based on requirements specifications? (K1)

A. White-box technique

B. Component testing

C. Black-box technique

D. Data driven testing

Solution

The syllabus considers structure-based approaches as white-box techniques. In contrast, the syllabus treats specification-based or experience-based approaches as black-box techniques.

Component testing involves testing an individual hardware or software unit, component, or module.

Data driven testing is a scripting technique which generally supports execution of capture or playback tools.

Answer C is correct.

References

ISTQB (2007a)1 p. 37; ISTQB (2007b)1 pp. 11 & 13; and IEEE Std 610.12-1990 (R2002) (2002), p. 14.

57

Sample Exam Questions: ISTQB Certified Tester Foundation Level

Sample Exam Questions: ISTQB Certified Tester Foundation Level

20. Which test design techniques should a tester use, respectively, to achieve the following: (a) check the documented features of the system, (b) ensure 100% decision coverage, and (c) detect likely defects and distribution? (K3)

Data driven testing is a scripting technique which generally supports execution of capture or playback tools, so choice A can be eliminated. The key term "likely" insinuates prior experience, whereas "exploratory" requires a tester to "explore" first and then adjust the test design based on the knowledge gained as part of the exploration. With regard to choice B, branch coverage is a form of structure-based technique.

A. Spedfkatlon-based, data-driven testing, and defect density techniques

B. Speoficatiorr-based, branch coverage, and exploratory

techniques "

C. Structure-based, equivalence partitioning, and exploratory techniques

D. Specification-based, structure-based, and experience-based techniques

Experience-based techniques generate test cases based on previous knowledge of the software. The term "likely" indicates prior knowledge of the system.

Answer D is considered correct.

References

ISTQB (2007a), p. 37; ISTQB (2007b), pp. 13-14 & 16; and logic.

Solution

Although presented in a K1 question format with the complexity of a K2, this question is really a K3 because it deals with practical application of multiple concepts.

Specification-based techniques create test cases from models (specifications) of the software.

Structure-based techniques derive test cases from the internal architecture (structure) of the software, thus allowing for coverage measurement.

It is also possible to answer the question using the process of elimination. Choice C can be eliminated because structure-based techniques rely on the internal architecture (structure) of the software. To simplify the choices, ignore "specification-based" because it is common to choices A, B, and D anyway.

58

59

Sample Exam Questions: ISTQB Certified Tester Foundation Level

21. What technique captures system requirements that contain logical conditions? (K1)

A. Boundary value

B. Equivalence partition

C. Decision table

D. State transition

.;:..:

Solution

An equivalence partition is a set of input or output values that will be processed Similarly by the system. For example, ages 13 to 19 years old are valid values for a teenager equivalence partition. Values less than 13 (too young) and values more than 19 years old (too old) belong to invalid teenager equivalence partitions.

Boundary values are the minimum and maximum values of an equivalence partition. Following the example above, 13 years old is the minimum boundary value and 19 years old is the maximum boundary value for the valid teenager equivalence partition.

Decision table testing identifies conditions and actions of the system. For example, an insurance application will perform a certain action (offer or decline coverage) based on the client's conditions (age, gender, driving history, etc.). Such conditions will be documented in a table to depict various combinations of conditions and the associated actions (calculations of premiums prior to offering or declining insurance coverage).

State transition testing focuses on how states (status or condition) transition from one status to the next depending on events and history. For example, individuals may go through several states such as Single, married, separated, divorced, and widowed.

Answer C is correct.

60

Sample Exam Questions: ISTQB Certified Tester Foundation Level

Reference

ISTQB (2007a), p. 38.

61

Sample Exam Questions: ISTQB Certified Tester Foundation Level

22. Input and output combinations that will be treated the same way by the system can be tested using which technique? (Kl)

A. Boundary value

B. Equivalence partition

C. Decision table

D. State transition

Solution

Please refer to the solution text of question 21.

Answer B is correct.

Reference

15TQB (2007a), p. 38.

62

Sample Exam Questions: ISTQB Certified Tester Foundation Level

23. Which test suite will check for an invalid transition using the diagram below? (K3)

A. 50 - 51- 52 - 53 - 51- 54

B. 50 - 51 - 54 - 51 - 52 - 53

C. SO - Sl - S3 - Sl - S2 - Sl ~ 50-51-52-53-51-52

Solution

5tate transition diagrams are depicted using several variations. A circle (or oval, square, rectangle, etc.)·generally represents a state. An arrow (or a tine) shows the transition from one state to the next. Sometimes, arrows are labeled (by an event) to show how a state transitions to the next state. For example, a death of a spouse (event) may be used as the label for an arrow to show how the event triggered the transition from married to widowed status.

Simply review each potential answer to determine if the states and transitions are valid based on the state transition diagram. Take note that the question above was looking for invalid transitions-there were two of such kind in choice C (51 to 53 skipped 52, and 52 to 51). The diagram states that 52 cannot transition to 51, but in reality that transition can be made.

Answer C is correct.

63

Sample Exam Questions: ISTQB Certified Tester Foundation Level

Reference

ISTQB (2007a), p. 38.

64

\

Sample Exam Questions: ISTQB Certified Tester Foundation Level

24. How are integration testing and use case testing similar and dissimilar? (K2)

A. Both checks for interactions: integration for components, use case for actors

B. Both are black-box techniques: integration is low-level, use case is high-level

C. Both are static testing: developers perform integration, users execute use case tests

D. Both are V&V techniques: integration is for validation, use case is for verification

Solution

The process of elimination can be effective when answering difficult questions that require comparing and contrasting two concepts, ideas, or techniques. Choice B can be eliminated because integration testing often requires familiarization with the structure of the codes, thus, a white-box technique.

From the V-model, static techniques are on the left-hand side of the V (verification), which includes walkthroughs, reviews, and inspections. Given this, choice C can be eliminated.

Choice D IS a plausible answer. However, the V-model, specifically the right-hand side (validation), makes the statement invalid for use case testing. Note that use case in and of itself (not use case testing) can be used for verification purposes (verifying requirements). The question stem asks for use case testing and not just use case.

Answer A is correct.

Reference

ISTQB (2007a), pp. 22 & 39.

65

}

} else if (p > q) { t = 5;,

Sample Exam Questions: ISTQB Certified Tester Foundation Level

Sample Exam Questions: ISTQB Certified Tester Foundation Level

25. How many test cases are needed to achieve 100% decision coverage? (K3)

if (p = q) {

S :: S + Ii

if (s < 5) {

t :: 10;

26, What analysis determines which parts of the software have been executed? (Kl)

A, Impact analysis

B. Code coverage

C. Gap analysis

D. Cyclomatic complexity

}

A. 3 B, 6

C. 5

D. 4

solution

Impact analysis assesses the level of required changes prior to implementing a change request.

Code coverage analyzes which parts of the code have been executed.

Solution

Gap analysis serves as a plausible distracter. However, the term is not mentioned in the syllabus or the glossary.

Remember that decision coverage is stronger than statement coverage: 100% decision coverage guarantees 100% statement coverage, but not vice versa.

Cyclomatic complexity counts the number of independent paths in a program.

In the question above, two test cases will ensure 100% statement coverage (aU statements or lines of codes will be executed).

Answer B is correct.

References



p = q and s < 5 p>q

ISTQB (2007a), p. 40 and ISTQB (2007b), pp. 10, 121 & 18.



To achieve 100% decision coverage, all decisions in the code must be tested. As such, two additional test cases are needed.

..

p -= q and s >= 5 p<q

..

Answer D is correct.

Reference

ISTQB (2007a), p. 40.

66

67

Sample Exam Questions: ISTQB Certified Tester Foundation Level

27. Based on the error guessing test design technique, which of the following will an experienced tester MOST LIKELY test in

calendar software? (K3) .

i. First two letters of the month, e.g., MA can represent March or May

ii. First letter of the dqy, e.g., T can mean Tuesday or

Thursday .

ili, Leap year

iv. Number of days in a month v . Three-dlqlt days and months

A. i, ii, lv, and v

B. iii and iv

C. i, ii, iii, and iv

D. i, n, and v

Solution

l, ii, iii, and iv are all reasonable test case candidates for a calendar software. Although months can be displayed in a threecharacter format (JAN, FEB, MAR, etc.), days and months are rarely, if ever, implemented in a three-digit format, Given this, choices A and D can be eliminated as valid choices. Choice B is correct but because it is incomplete, it is inferior to choice C.

Answer C is correct.

References

ISTQB (2007a), p, 41; ISTQB (2007b), p. 16; and experience.

68

Sample Exam Questions: ISTQB Certified Tester Foundation Level

28. Which input combinations will a knowledgeable tester MOST UKEL Y use to uncover potential errors when testing a surname field? (K3)

A. Johnson, de la Cruz, and Morgan

B. Go, Stephanopoulous, and Venkatsewaran

C. Srnit, Smyth, and Smithsonian

D. O'Brien, Zeta-Jones, and Young Pow

Solution

The use of surnames with more than one word (de la Cruz) in choice A has its merits. However, using both Johnson and Morgan is redundant and offers no additional benefit.

Choice B tries to test a potential reserve word (Go) and unusually long surnames. Similar to choice A, Stephanopoulous and Venkatsewaran are redundant.

Although Smit and Smithsonian are excellent test data (Smit can potentially return a large result induding Smith, Smiths, Smithsonian, etc.), Smyth offers little benefit in a normal application. It will, however, be excellent for applications that use phonetics.

Lastly, choice D test for surnames with problematic elements such as the apostrophe in O'Brien, the dash in Zeta- Jones, and the space and special character in Young Pow. Given this, choice D is far superior to the other choices.

Answer D is considered correct.

References

ISTQB (2007a), p. 41; ISTQB (2007b), p. 16; and experience.

69

Sample Exam Questions: ISTQB Certified Tester Foundation Level

Sample Exam Questions: ISTQB Certified Tester Foundation Level

29. Which of the following demonstrates independence in testing? (K3)

30. Which of the following is a KEY task of a tester? (K1)

J. Independent testers are external to the organization

K. Independent testers are part of the development team

L. Independent testers are from the user community

M. Programmers who wrote the code serve as independent testers

N. Customers who wrote the requirements serve as independent testers

A. Reviewing tests developed by others

B. Writing a test strategy for the project

C. Deciding what should be automated

D. Writing test summary reports

Solution

A. J, L, and N

B. J, K, L, and N

C. K, M, and N

D. Jr L, M, and N

The syllabus discusses two test roles: test leader (sometimes called test manager or test coordinator) and tester. In general terms, the test leader plans and manages the testing processes. In contrast, the tester performs the actual work.

Choices Band C deals with strategy and decisions, which are test leader tasks. For choice Dr gathering data is a task for testers, but the preparation of a summary report is not.

Solution

Programmers cannot objectively test their own codes. Because choices C and 0 incfude statement Mr both choices can be ruled out as valid choices. Statement K distinguishes choice A from B. Given that statement K is valid, we can deduce that choice B is a superior choice.

Answer A is correct.

70

Reference

ISTQB (2007a), pp. 45-46.

Answer B is considered correct.

Reference

ISTQB (2007a), p. 45.

71

Sample Exam Questions: 15TQB Certified Tester Foundation Level

31. In software testing, what is the MAIN purpose of exit criteria? (K1)

A. To enhance the security of the system

B. To prevent endless loops in codes

C. To serve as an alternative or "Plan B"

D. To define when to stop testing

Solution

Exit criteria define when to stop testing. Factors to consider when defining an exit criterion include code coverage, reliability metrics, technical and business risks, and triple constraints (scope, time, and cost).

Answer D is correct.

References

I5TQB (2007a), p. 47 and I5TQB (2007b), p. 16.

72

Sample Exam Questions: ISTQB Certified Tester Foundation Level

32. Which test approaches or strategies are characterized by the descriptions below? (K2)

5. Analytical approaches

T. Model-based approaches

U. Methodical approaches

V. Consultative approaches

1. Relies on guidelines from domain experts

2. Includes error guessing and fault-attacks

3. Uses statistical information about failure rates

4. Focuses on areas of greatest risk

A. S4, T3, U2, Vl

B. 51, T2, U3, V4

C. 52, T3, Ul, V4

D. 53, T4, U2, VI

Solution

Analytical approaches (5) direct testing based on greatest risks (4).

Model-based approaches (T) capitalize on failure rate statistics (3).

Methodical approaches (U) include error guessing and faultattacks (2).

Consultative approaches (V) leverage the knowledge of others (1).

Answer A is correct.

Reference

15TQB (2007a), p. 48.

73

Sample Exam Questions: ISTQB Certified Tester Foundation Level

33. Which of the foHowing can be used to measure progress against the exit criteria? (K2)

W. Number of test cases that passed or failed

X. Number of defects found in a unit of code

Y. Dates for milestones and deliverables

Z. Subjective confidence of testers in the product

A. WI X, V, and Z B W, X, and Y

C. Wand X

D. W, X, and Z

Solution

Exit criteria define when to stop testing. Factors to consider when defining an exit criterion include code coverage, reliability metrics, technical and business risks, and triple constraints (scope, time, and cost), to name a few.

Statements W, X, and Yare all valid. Others may argue that statement Z is not an acceptable exit criterion because of its qualitative (subjective) tangent; however, the syllabus stated that subjective confidence of testers in the product is a common test metric.

Answer A is considered correct.

References

ISTQB (2007a), p. 49 and ISTQB (2007b), p. 16.

74

Sample Exam Questions: ISTQB Certified Tester Foundation Level

34. What type of risk includes potential failure areas in the software? (K1)

A. Project risks

8. Product risks

C. Economic risks

D. Requirements risks

solution

Project risks encompass uncertainties associated with the project's capabilities to meet its intended objective. In a related context, the project will eventually deliver a product (software). Deficiencies in the software are therefore potential product risks. Economic risks entail financial implications of uncertainties, while requirements risks include misunderstood, unclear, or incomplete documentation of customer needs.

Answer B is correct.

Reference

ISTQB (2007a), p. 52.

75

Sample Exam Questions: ISTQB Certified Tester Foundation level

35. Based on the IEEE Standard for Software Test Documentation (IEEE Std 829-1998), in which sections of the test incident report should the following items be recorded? (K3)

Sections

a) Test incident report identifier

b) Summary

c) Incident description

d) Impact

Items

1. Unique identifier

2. Version level of the test items

3. Inputs

4. Expected results

5. Actual results

6. Anomalies

7. Date and time

A. a: 1; b: 2 and 7; c: 3, 4, and 5; d: 6

B. a: 1; b: 6 and 7; c: 3, 4, and 5; d: 7

C. a: 1; b: 2; c: 3, 4, 5, 6, and 7

D. a: 1; b: 6 and 7; c: 3, 4, and 5

Solution

Four sections ofthe test incident report are listed as a, b, c, and d, and it is reasonable to assume that they correct. Given this, we need to map items 1 to 7 to their respective sections.

Item 1 (unique identifier) dearly belongs to section a (test incident report identifier), Unfortunately, it is not helpful in eliminating the other choices, Items 2 to 7 do not appear to belong to section d, so choices A and B can be ruled out. A summary of anomalies (item 6) can potentially be included in section b (summary); however, item 7 (date and time) seems out of place in the summary. It appears plausible that items 3 to 7 can be documented in section c (incident descrlption).

76

Sample Exam Questions: ISTQB Certified Tester Foundation level

Answer C is considered correct.

References

ISTQB (2007a), p. 54 and IEEE (1998), pp, 13-14.

77

Sample EXam Questions: ISTQB Certified Tester foundation Level

36. Based on the IEEE Standard for Software Test Documentation (IEEE Std 829-1998), which of the following sections are part of the test summary report? (K2)

a) Test summary and report identifier

b) Summary

c) Variances

d) Anomalies

e) Comprehensive assessment

f) Approvals

A. a, b, e, and f

B. a, b, c, d, and f

C. a, b, c, e, and f D, a, b, c, and f

Solution

Sections of the test summary report include the following:

a) Test summary report identifier

b) Summary

c) Variances

d) Comprehensive assessment

e) Summary of results

f) Evaluation

g) Summary of activities

h) Approvals

Anomalies are listed as part of the incident description in the test incident report, making option B incorrect.

Although options A and D are correct, option C is more inclusive making is a superior choice. r

Answer C is considered correct.

References

ISTQB (2007a), p. 54 and IEEE (1998), p. 14.

78

Sample Exam Questions: ISTQB Certified Tester Foundation Level

37. What is the name of a skeletal implementation of a software component that is used for testing? (K1)

A. Use case B, Domain

C. Driver

D. stub

Solution

A use case is a sequence of transactions in a dialogue between a user and the system with a tangible result.

Domain is a set from which valid input and/or output values can be selected.

Driver is a software component or test tool that replaces a component that takes care of the control and/or the calling of a component or system.

Stub is a special-purpose skeletal implementation of a software component with a primary objective of replacing an actual component for testing purposes,

Answer D is correct.

References

ISTQB (2007a), p. 57 and ISTQB (2007b), pp. IS, 29, & 35.

79

Sample Exam Questions: ISTQB Certified Tester Foundation Level

38. Which of the following are potential benefits of using test support tools? (K2)

A. Ensuring greater consistency and minimizing software project risks

B. Reducing repetitive work and gaining easy access to

test information ;'

C. Performing objective assessment and reducing the need for training

D. Allowing for greater reliance on the tool to automate the test process

Solution

Watch out for the word AND in most potential answers. For choice A, ensuring greater consistency is a potential benefit of using test support tools. However, the second portion of choice A is too esoteric to be considered a potential benefit. A similar issue is also evident in the second portion of choice C. USing common sense, choice D can be ruled out as a valid choice, Given this, choice B is considered correct.

Answer B is considered correct.

Reference

ISTQB (2007a), p. 57.

80

Sample Exam Questions: ISTQB Certified Tester Foundation Level

39. Which test support tool can be used to enforce coding standards? (K1)

A. Static analysis tool

B. Performance testing tool

C. Test comparator

D. Test management tool

Solution

Developers, testers, and QA staff use static analysis tools before dynamic testing to enforce coding standards, analyze structures, verify dependencies, and understand the code.

Performance testing tools help simulate increased load, volume, and stress on the system to monitor and report the behavior of the system under intense conditions.

Test comparators determine differences across data, flies, database, and results.

Test management tools aid in the administration of test processes such as defect tracking, version control, traceability, and report generation.

Answer A is correct.

Reference

ISTQB (2007a), pp. 57-60.

81

Sample EXam Questions: ISTQB Certified Tester Foundation Level

40. What should be considered when introducing a tool into an organization? (Kl)

A. Assessing the organizational maturity

B. Counting the number of systems to be tested

C. Calculating the ratio between programmers and testers

D. Reviewing the exit criteria of previous projects

Solution

Key principles of introducing a tool into an organization indude:

• assessing the maturity of the organization,

• evaluating against objective requirements and criteria,

• conducting proof-of-concept tests to ascertain suitability,

• evaluating the vendor from various perspectives, and

• identifying internal training and coaching needs.

Answer A is correct.

Reference

ISTQB (2007a), p, 64.

82

Chapter 4: Practice Questions B

."."c.'."N' •• 'N.N.W.W •••• ".N.'m •• ' •• W.NU~,_".U ••••• LU~L_~C •••••• __ w •• w •• , •• w." •• ,.w._, •• , •• , •••.•• ~_., •• ~, .. , •• ''" •• ,.w_w._ .. w.P.w._.w •• _"." •••• , •• , •• '.~.n.''.''~.w_ •• _. __ • __ • __ ••• , ••••• "w ••• ·~

1. Which factors contribute to humans making mistakes that can lead to faulty software?

1. Setting aggressive schedule II. Integrating complex systems III. Allocating adequate resources IV. Failing to control changes

A. I and II are true; III and IV are false

B. II and IV are true; I and III are false

C. I, II and IV are true; III is false

D. II II and III are true; IV is false

2. How can software defects in future projects be prevented from reoccurring?

A. Creating documentation procedures and allocating resource contingencies

B. Asking programmers to perform a thorough and independent testing

c. Combining levels of testing and mandating inspections of all documents

D. Documenting lessons learned and determining the root cause of problems

3. Which of the following are USUALLY stated as testing objectives?

I. Finding defects in the software II. Reducing maintenance costs

III. Confirming that the system works IV. Assessing the quality of the software V. Meeting schedule milestones

A. I and II

B. I, III, and IV

C. II, IV, and V

D. III and IV

83

Sample Exam Questions: ISTQB Certified Tester Foundation Level

4. Which test approach uses all combinations of input values and preconditions?

A. Component testing

B. Error guessing

C. Keyword driven testing

D. Exhaustive testing

5. Which of the following is a KEY test control task?

A. Initiating corrective actions

B. Determining the scope

C. Implementing the test policy

D. Scheduling test implementation

6. Which of the following is a MAJOR task when evaluating the exit criteria?

A. Creating test suites and cases for efficient execution

B. Writing a test summary report for stakeholders

C. Handing the testware to the maintenance organization

D. Identifying any required infrastructure and tools

7. What principles do "avoiding author bias" and "communicating problems constructively" represent?

A. Preventive testing and reactive testing

B. Experience-based testing and interoperability testing

C. Independent testing and good interpersonal skills

D. Criticism avoidance and effective relationships

8. Which test may OPTIONALLY be included in the common type of the V-model?

A. Component (unit) testing

B. Acceptance testing

C. System integration testing

D. Validation and verification

84

Sample Exam Questions: ISTQB Certified Tester Foundation Level

9. What is the difference between component testing and integration testing?

'.

A. Component testing tests interfaces; integration testing searches for defects

B. Component testing searches for defects; integration testing tests interfaces

C. Developers perform component testing; testers perform integration testing

D. Testers perform component testing; users perform integration testing

10. Which test investigates both functional and non-functional system requirements?

A. Alpha testing

B. System testing

C. Acceptance testing

D. Confirmation testing

11. Which of the following can be tested as part of operational testing?

A. Component interaction

B. Probe effect

C. State transition

D. Disaster recovery

12. What is the KEY difference between black-box and white-box testing?

A. Black-box is functional; white-box is structural

B. Black-box is functional; white-box is non-functional

C. Black-box has a wider statement coverage than white-box

D. Black-box can only be performed after white-box

85

Sample Exam Questions: ISTQB Certified Tester Foundation Level

13. Which test is usually run many times and generally evolve slowly?

A. Performance testing

B. Stress testing

C. ReJiability testing

D. Regression testing

14. Which defects are OFTEN much cheaper to remove?

A. Usability defects found by customers

B. Defects in infrequently used functionality

C. Defects that were detected early

D. Minor defects that were found by users

15. Which statements correctly describe certain phases of a formal review?

A. Looking for defects occurs during kick-off phase Fixing defects found happens during rework phase

B. Personnel selection OCcurs during planning phase

Gathering metrics happens during the review meeting phase

C. Distributing documents occurs during the planning phase Personal review happens during individual preparation phase

D. Personnel selection occurs during planning phase Rxing defects found happens during rework phase

16. Which defect can typically be discovered using a static analysis tool?

A. Inconsistencies in numerical calculations

B. Programming standards violations

C. Problems related to system usability

D. Internal and external system reliability

Sample Exam Questions: ISTQB Certified Tester Foundation Level

17. What consists of a set of input values, execution preconditions and expected results?

A. Test script

B. Test procedure specification

C. Test case

D. Test data

18. Which documents specify features to be tested, approach, and pass/fail criteria?

A. Test plan and test design specification

B. Test plan and test case specification

C. Test procedure specification and test design specification

D. Test case specification and test procedure specification

19. Which technique is appropriate to test changes on old and undocumented functionalities of a system?

A. Specification·based technique

B. Black-box technique

C. White·box technique

D. Data driven testing technique

20. Which test design technique relies heavily on prior thorough knowledge of the system?

A. Data driven testing technique

B. Experience-based technique c. Whit~box technique

D. structure-based technique

21. Which set of test data demonstrates equivalence partitioning to check whether a customer is a teenager or not?

A. 10, 15 and 19 years

B. 13, 19 and 25 years

C. 13, 16 and 19 years

D. 12, 13 and 20 years

86 87

Sample Exam Questions: ISTQB Certified Tester Foundation Level

22. What technique would be M05T appropriate to check status changes based on certain events?

A. 5tate transition

B. Equivalence partitioning

C. Boundary value analysis

D. Decision table

23. Using the diagram below, which test suite will check for ALL . valid state transitions using the LEAST effort?

A. 50 - 51 - 52 - 54 - 51 - 54 - 51 - 52 "' 53 - 51

B. 50 -51- 52 - 54 - S1 - 52 - 53 - S1

C. 50 - 51 - 54 - 51 - 52 - 53 - 51

D. 50 - 51 - 52 - 54 - 51 - 54 - 51 - 52 - 53

88

Sample Exam Questions: ISTQB Certified Tester Foundation Level

24. Using the diagram below, which test suite will uncover invalid state transitions for employee status reporting software?

A. Prospective - Active - Resigned - Active - Terminated - Purged

B. Prospective - Active - On Leave - Active - Resigned - Retired

C. Prospective - Active - Retired - Active - On Leave - Purged

D. Prospective - Active - On Leave - Active - Retired - Active

25. Which of the following assertions about code coverage are correct?

A. Statement coverage usually requires more test case suites

B. 100% statement coverage guarantees 100% decision coverage

C. 100% decision coverage implies 100% statement coverage

D. Decision tables cannot be used to list statement coverage values

89

Sample Exam Questions: ISTQB Certified Tester Foundation Level

26. How many test cases are needed to achieve 100% statement coverage?

if «temperature < 0) or (temperature> 100» { <;tl~rt (" DANGER" ) ;

~f «speed> 100) and (load <= 50» { speed'" 50;

}

} else {

check"" false;

}

A. 5

B. 4

C. 2

D. 3

27. Using an error guessing test design technique to convert temperature (Celsius to Fahrenheit, and Fahrenheit to Celsius), experienced testers will MOST LIKELY use which set of test data?

A. -1, 0, 89.6 and 212

B. -40,37.78, and 100 C -1,0, 1 and 37.78 D, -40, 0, 32 and 100

28. By creating future tests based on the results of previous tests, a tester is demonstrating what type of informal test design technique?

A. Security testing

B. Non-functional testing

C. Exploratory testing

D. Interoperability testing

90

Sample Exam Questions: ISTQB Certified Tester Foundation Level

29. Which of the following are potential drawbacks of independence in testing?

01. Independent testers may feel they are not part of the development team

02, Developers may lose a sense of personal responsibility for quality

03~ Project managers will not have as much control on the project

04, Customers may end up requesting features that are technically impossible

A. 01 and 02

B. 01, 02 and 03

C. 03 and 04

O. 01, 02, 03 and 04

30. Which tasks are performed by a test leader versus a tester?

S. Writing a project test strategy

T. Selecting tools to support testing U, Preparing and acquiring data

V. Scheduling tests

A. Test leader: 5 and V; Tester: T and U

B. Test leader: 5, T and V; Tester: U

C. Test leader: S, U and V; Tester: T O. Test leader: S; Tester: T, U and V

31. Which document specifies the sequence of test executions?

A. Test procedure specification 8. Test design specification

C. Test case specification

O. Test plan

91

Sample Exam Questions: ISTQB Certified Tester Foundation Level

32. Stochastic testing is an example of which test approach or strategy?

A. Model~based

B. Analytical

C. Methodical

D. Heuristic

33. Which sections are included as part of the test summary report?

W. Variances

X. Comprehensive assessment

Y. Evaluation

Z. Summary of activities

A. W, X and Y

B. W, X, Y and Z

C. Wand X

D. W, X and Z

34. Which is a potential product risk factor?

A, Failure of third party vendor

B. Training issues

C. Problems requirements definition

D. Poor software functionality

92

Sample Exam Questions: ISTQB Certified Tester Foundation level

35. Based on the IEEE Standard for Software Test Documentation (IEEE Std 829-1998), which sections of the test incident report should the following items be recorded?

Sections

a) Test incident report identifier

b) Summary

c) Incident description

d) Impact

Items

1. Impact on test plans

2. Unique identifier 3, Anomalies

4. Procedure step

5. Environment

6. References to other relevant documents

A. a: 2; b: 4; c: 1, 3 and 5; d: 6 B, a: 2; b: 3; c: 4, 5 and 6; d: 1

C. a: 2; b: 6; c: 3, 4 and 5; d: 1

D. a: 2; b: 1; c: 3,4 and 5; d: 6

36. Based on the IEEE Standard for Software Test Documentation (IEEE Std 829~1998), which of the following sections are part of the test summary report?

a) Test summary and report identifier

b) Comprehensive assessment

c) Summary of results

d) Evaluation

e) Observers

f) Approvals

A. a, b, c, d and e B, a, b, c, e and f

C. a, c, d, e and f

D. a, b, c, d and f

93

Sample Exam Questions: ISTQB Certified Tester Foundation Level

37. What is the name of a temporary software component that is used to call another component for testing purposes?

A. Domain

B. Use case

C. Stub

D. Driver

38. Which of the following is a potentialrlsk in using test support tools?

A. Underestimating the effort needed to maintain the test assets

B. losing access to important testing information when needed

C. Relying too much on qualitative and quantitative assessments

D. lowering the morale of the test team because of repetition

39. How are (a) static analysis tools and (b) performance testing tools different?

A. (a) helps in enforcing coding standards; (b) tests system performance

B. (a) analyzes security vulnerabilities; (b) measures the effectiveness of test cases

C. (a) prepares codes prior to testing; (b) prepares codes prior to stress testing

D. (a) highlights unreachable conditions; (b) improves system performance

94

Sample Exam Questions: ISTQB Certified Tester Foundation Level

40. Which of the following is a potential pilot project objective when introducing a test support tool into an organization?

A. Measuring the satisfaction of management for staying within scope

B. Assessing whether the benefits will be achieved at

reasonable cost

C. Receiving compliments from the users on the aesthetic

aspects of the tool

D. Reducing the amount of overtime need to finish the project

on time

95

Sample Exam Questions: ISTQB Certified Tester Foundation Level

Correct Answers and Cognitive Levels

1 C K2
2 D K1
3 B K2
4 D K1
5 A Kl
6 B Kl
7 C K2
8 C K1
9 B K2
10 B K1
11 D K1
12 A K2
13 D K1
14 C Kl
15 D K2
16 B K1
17 C K1
18 A K2
19 C K3
20 B K1 . .;

21 D K3
22 A K1
23 A K3
24 B K3
25 C K2
26 C K3
27 D K3
28 C K1
29 A K2
30 B K3
31 A K1
32 A Kl
33 B K2
34 D K3
35 C K2
36 D K1
37 D Kl
38 A Kl
39 A K2
40 B Kl 96

1. which factors contribute to humans making mistakes that can lead to faulty software? (K2)

1. Setting an aggressive schedule II. Integrating complex systems III. Allocating adequate resources IV. Failing to control changes

A. I and II are true; III and IV are false

B. II and IV are true; I and III are false

C. I, II, and IV are true; III is false D, 1/ II, and III are true; IV is false

Solution

The factors that contribute to humans making mistakes that can lead to faulty software include:

• aggressive timeline,

• inadequate resources,

complicated code,

• complex infrastructure,

• rapid technological changes,

• multiple interdependencies,

• environmental conditions (radiation, pollution, etc.), and

• lack of change management process.

Answer C is correct.

Reference

ISTQB (2007a), p. 11.

97

Sample Exam Questions: ISTQB Certified Tester Foundation Level

2. How can software defects in future projects be prevented from reoccurring? (Kl)

A. Creating documentation procedures and allocating resource contingencies

B. Asking programmers to perform a thorough and independent testing

C. Combining levels of testing and mandating inspections of aU documents

D. Documenting lessons learned and determining the root ca use of problems

Solution

Watch out for the word AND in most choice options--oorr, sides must be correct. For choice At the first portion is correct but the second portion indirectly addresses the question. Choice B is somewhat contradictory. It will be difficult for persons who

wrote the code (programmers) to be objective when testing their own code. Combining levels of testing (for example, combining system testing and acceptance testing) is generally not recommended because each test level has its own set of unique test objectives.

Answer D is correct.

References

ISTQB (2007a), p. 12 and experience.

98

Sample Exam Questions: ISTQB Certified Tester Foundation Level

3. Which of the following are USUALLY stated as testing objectives? (K2)

1. Finding defects in the software II. Reducing maintenance costs

III. Confirming that the system works IV. Assessing the quality of the software V. Meeting schedule milestones

A. I and II

B. I, III, and IV

C. II, IV, and V

D. III and IV

Solution

Common test objectives include finding defects, gaining confidence about the quality of the software, and preventing defects just to name a few. Statement II is a valid test objective; however, statement II is included only in choices A and C. Choice A is incomplete because statements III and IV were not listed. Likewise, choice D is missing statements I and II. In contrast, choice C includes statement V, which is more appropriate as a project objective instead of a test objective. Although choice B is incomplete, it is more accurate than choices A and D.

Answer B is considered correct.

Reference

ISTQB (2007a), p. 13.

99

Sample Exam Questions: ISTQB Certified Tester Foundation Level

4. Which test approach uses all combinations of input values and preconditions? (Kl)

A. Component testing

B. Error guessing

C. Keyword driven testing

D. Exhaustive testing "~

Solution

Component testing involves testing an individual hardware or software unit, component, or module.

Error guessing relies on the tester's experience to anticipate potential defects and would therefore involve designing tests to specifically expose those defects.

Keyword driven testing is a scripting technique which uses data files that contain test data, expected results, and appficationspecific keywords. Control scripts call special supporting scripts to interpret the keywords.

Exhaustive testing (complete testing) tests all combinations of input values and preconditions.

Answer D is correct.

References

ISTQB (2007a), p. 14 and ISTQB (2007b), pp. 11, 16, & 20.

100

Sample Exam Questions: ISTQB Certified Tester Foundation Level

5. Which of the following is a KEY test control task? (Kl)

A. In'itiating corrective actions

B. Determining the scope

C. Implementing the test policy

D, Scheduling test implementation

Solution

The fundamental test control tasks are:

.. measuring and analyzing results,

.. monitoring and documenting progress,

.. ensuring proper test coverage,

.. adhering to the exit criteria,

• initiating corrective actions, and

.. making decisions.

Determining the scope, implementing the test policy, and scheduling test implementation are all test planning tasks.

Answer A is correct.

References

ISTQB (2007a), p. 15 and experience ..

101

Sample Exam QuestkJns: ISTQB Certified Tester Foundation Level

6. Which of the following is a MAJOR task when evaluating the exit criteria? (K1)

A. Creating test suites and cases for efficient execution

B. Writing a test summary report for stakeholders

C. Handing the testware to the maintenance organization

D. Identifying any required infrastructure and tools

Solution

The following are the major tasks involved in evaluating the exit criteria:

..

checking test logs against the exit criteria specified in the test planning,

assessing if more tests are needed or if the exit criteria specified should be changed, and

writing a test summary report for stakeholders.

..



Response options A, C, and D are tasks in test implementation and execution/ test closure activities, and test analysis and desiqn, respectively.

Answer B is correct.

References

ISTQB (2007a), p. 16.

102

Sample Exam Questions: ISTQB Certified Tester Foundation Level

7. What principles do "avoiding author bias" and "communicating problems constructively" represent? (K2)

A. Preventive testing and reactive testing

B. Experience~based testing and interoperability testing

C. Independent testing and good interpersonal skills

D. Criticism avoidance and effective relationships

Solution

The process of elimination can help narrow down the answer for this question. For example, "reactive testing" in response option A and "interoperability testing" in response option B have nothing to do with "communicating problems constructively." Response options C and 0 are both reasonable answers. However, "avoiding author bias" means separation of duty (independence) between the author (programmer) and the reviewer (tester) .

Answer C is correct.

Reference

ISTQB (2007a), p. 17.

103

Sample Exam Questions: ISTQB Certified Tester Foundation Level

8. Which test may OPTIONALLY be included in the common type of the V-model? (Kl)

A. Component (unit) testing

B. Acceptance testing

C. System integration testing

D. Validation and veriticatlon

Solution

The common levels of testing include unit, integration, system, and acceptance testing. Response options A and B are normally included in the V-model and therefore cannot be considered as optional. Response option D deals more with both sides of the Vmodel. System integration testing is a hybrid of integration testing and system testing-an "uncommon" test level-thus making it a plausible answer.

Answer C is correct.

Reference

ISTQB (2007a), p. 20.

104

Sample Exam Questions: ISTQB Certified Tester Foundation Level

9. What is the difference between component testing and integration testing? (K2)

A. Component testing tests interfaces; integration testing searches for defects

B. Component testing searches for defects; integration testing tests interfaces

C. Developers perform component testing; testers perform integration testing

D. Testers perform component testing; users perform integration testing

Solution

Component testing is defined as the process of searching for defects in, and verifying the functions of, software units (l.e., modules, programs, objects, classes, etc.) individually. Typically, component testing is done by the developer or programmer.

Integration testing is defined as the process of testing the interface between components. Here the components are combined and tested as a group to verify if each component is interfacing and/or interacting correctly. This test can be performed by either developers or testers.

Answer B is correct.

Reference

ISTQB (2007a), p. 22.

105

Sample Exam Questions: ISTQB Certified Tester Foundation level

10. Which test investigates both functional and non-functional

system requirements? (Kl)

A. Alpha testing

B. System testing c. Acceptance testing

D. Confirmation testing ?

SOlution

Alpha testing is defined as a simulated or actual operational testing by potential users/customers or an independent test team at the developers' site, but outside the development organization. Alpha testing is often employed for off-the-shelf software as a form of internal acceptance testing.

System testing is defined as the process of testing the whole system to verify if it meets the specified requirements and behaves as defined by the scope of a development project. It investigates both functional and non-functional requirements of the system.

Acceptance testing is defined as the process of testing with respect to the users' or customer's requirements and interests. It investigates whether or not the system satisfies the acceptance criteria and enables the user, customer' or other authorized entity to determine whether or not to accept the system.

Confirmation testing or re-testing is defined as running the test cases that failed the last time they were run so as to verify the success of corrective actions or fixes that were made.

Answer 8 is correct.

References

ISTQB (2007a), p. 23 and ISTQ8 (2007b), pp. 6-7, 12, & 29.

106

Sample Exam Questions: ISTQB Certified Tester Foundation level

11. Which of the following can be tested as part of operational testing? (Kl)

A. Component interaction

B. Probe effect

C. State transition

D. Disaster recovery

Solution

Operational testing can include the following:

• testing of backup/restore,

• disaster recovery,

• user management,

• maintenance tasks, and

• periodic checks of security vulnerabilities.

Response options A, 8, and C are part of component testing and/or integration testing.

Answer D is correct.

Reference

ISTQB (2007a), p. 24.

107

Sample Exam Questions: ISTQB Certified Tester Foundation Level

12. What is the KEY difference between black-box and white-box

testing? (K2) .

A. Black-box is functional; white-box is structural

B. Black-box is functional; white-box is non-functional

C. Black~box has a wider statement coverage than white-box

D. Black-box can be performed only after white-box

.;:.

Solution

Black-box testing is a testing technique whereby the internal structure of the component or system being tested is not known by the tester. These tests can be functional or non-functional and are applicable to all levels of software testing: unit/component, integration, functional, system, and acceptance testing. While black-box testing can uncover unimplemented parts of the specification, it does not guarantee that aU existent paths are tested.

White-box testing is a testing technique that allows tester to "peek" inside the "box." It focuses specifically on using the internal structure of the component or system being tested. Tests generated based on this technique incorporate coverage of the written code, branches, paths, statements, and internal logic of the code.

A careful review of the response options will help eliminate B, C, and D. Black-box testing can be functional or non-functional, making response option B incorrect. Response option C is incorrect as well since it is the white-box testing that goes through every code statement and all branches and paths. By doing so; it has wider statement coverage. There is no order as to which test technique should be performed first. The word "only" in response option D makes it incorrect

Answer A is correct.

Reference .

ISTQB (2007a), p. 25.

108

Sample Exam Questions: ISTQB Certified Tester Foundation Level

13. Which test is usually run many times and generally evolve slowly? (K1)

A. Performance testing

B. Stress testing

C. Reliability testing

D. Regression testing

solution

Performance testing is a type of testing performed to determine how fast (or slow) the system performs under an acceptable workload. Its purpose is to define and identify bottlenecks in the system.

Stress testing is a type of testing performed to evaluate the stability of a system. It involves testing the system at or beyond

its operational capacity or specified requirements. -

Reliability testing is a type of testing performed to determine if a system can continue running significant load for an extended period of time or for an extended number of operations,

Regression testing is a type of testing performed to ensure that defects were not introduced to a previously tested system after it was modified or changed. It is performed every time the system or its environment has changed and generally evolves slowly.

Answer D is correct.

References

ISTQB (2007a), pp. 25-26 and ISTQB (2007b), pp. 23, 25, & 29.

109

Sample Exam Questions: ISTQB Certified Tester Foundation Level

14. Which defects are OFTEN much cheaper to remove? (Kl) A. Usability defects found by customers

B, Defects in infrequently used functionality

C. Defects that were detected early

D. Minor defects that were found by users

Solution

Key option C follows the principle that defects detected early are easier and less expensive to fix. Conversely, defects found at a later stage of the software development life cycle require a more in-depth analysis and therefore cost more. Response options A and D fall in the latter category. Just because the functionality in response option B is not used frequently does not necessarily indicate that it will be cheaper to fix.

Answer C is correct.

Reference

ISTQB (2007a), p, 29.

110

Sample Exam Questions: ISTQB Certified Tester Foundation Level

15. Which statements correctly describe certain phases of a formal review? (K2)

A. looking for defects occurs during the kick-off phase Fixing defects found happens during the rework phase

B. Personnel selection occurs during the planning phase Gathering metrics happens during the review meeting phase

C. Distributing documents occurs during the planning phase Personal review happens during the individual preparation

phase .

D. Personnel selection occurs during the plannmg phase Fixing defects found happens during the rework phase

Solution

The tasks for each main phase of a formal review are as follows:

• Planning Phase

o select personnel,

o allocate roles,

o define entry and exit criteria, and

o select which parts of documents to look at. e Kick-off Phase

o distribute documents,

o explain objectives, process, and documents to participants, and

o check entry criteria.

• Individual Preparation Phase

o note potential defects, questions, and comments.

• Review Meeting Phase

o log defects,

o produce recommendations for handling defects, and

o make decisions about the defects.

• Rework Phase o fix defects.

• Follow-up Phase

o check that defects have been addressed or resolved,

o gather metrics, and

o check or verify on exit criteria.

111

Answer 0 is correct.

Sample Exam Questions: ISTQB Certified Tester Foundation Level

Reference

ISTQB (2007a)! p. 3~.

112

Sample Exam Questions: ISTQB Certified Tester Foundation Level

16. Which defect can typically be discovered using a static analysis tool? (Kl)

A. Inconsistencies in numerical calculations

B. Programming standards violations

C. Problems related to system usability

D. Internal and external system reliability

Solution

Static analysis tools are tools used to analyze requirements or source code and identify mistakes or errors. They do not require that you execute the system under analysis,

A review of the response options will help eliminate A, C, and D. These response options will require you to run or execute the system for them to be discovered, thus making response option B the viable answer.

Answer B is correct.

Reference

ISTQB (2007a), p. 33.

113

Sample Exam Questions: ISTQB Certified Tester Foundation level

17. What consists of a set of input values, execution

preconditions; and expected results? (K1)

A. Test script

B. Test procedure specification

C. Testcase

D. Test data

Solution

Test script is defined as the detailed instructions for the execution and evaluation of results for a given test case. It defines the actions and pass/fail criteria.

Test procedure speottcatton is defined as the document that contains the test script.

Test case is defined as a set of test data (or input values), test scripts (or execution conditions), and expected results developed for the purpose of verifying one or more system requirements.

Test data is defined as a set of data (existing or new) used for testing new or revised systems or parts of the system.

Answer C is correct.

References

ISTQB (2007a), p, 36 and ISTQB (2007b), pp. 30-31 & 33-34.

114

Sample Exam Questions: ISTQB Certified Tester Foundation level

18. Which documents specify features to be tested, approach,

and pass/fail criteria? (K2)

A. Test plan and test design specification

B. Test plan and test case specification

C. Test procedure speclfication and test design spedflcatlon

D. Test case specification and test procedure specification

Solution

Test case specification is a document that contains a set of test cases which specifies the input values, expected results, and execution conditions for a test item.

Test design specification is a document that contains the test conditions or coverage items for a test item. Likewise, it specifies the refinements of the test approach; high level test cases, and pass/fail criteria.

Test ptan is a record of the test planning process. It is a document that describes the detail of the testing effort. These details consist of scope, approach, resources, and schedule of testing activities. Also, it identifies the test items (including the pass/fail criteria), features to be tested, th~ testing tasks, ~ho will be responsible for each task, and any risks that necessitate contingency planning.

Test procedure specification is a document that desc~ibes how a tester will run a test. It specifies the sequence of actions or steps to be taken by the tester in order to execute the test. Test procedure specification is also known as test script or manual test script.

Answer A is correct.

References

ISTQB (2007a), p. 36 and ISTQB (2007b)r pp, 30-33.

115

Sample Exam Questions: ISTQB Certified Tester Foundation Level

19. Which technique is appropriate to test changes on old and

undocumented functionalities of a system? (K3)

A. Specification-based technique

B. Black-box technique

C. White-box technique

D. Data driven testing technique

Solution

This is an example of a question that can be best answered using the process of elimination. Specifically, specification-based technique and black-box technique are synonymous and can be eliminated as potential answers because there should be one and only one correct answer. Data-driven testing technique documents test inputs and expected results for automated or scripted test execution. Therefore, white-box technique is the closest answer to the question because test cases can be derived based on existing code.

Answer C is correct.

References

ISTQB (2007a), p. 37 and ISTQB (2007b), p. 13.

116

Sample Exam Questions: ISTQB Certified Tester Foundation Level

20. Which test design technique relies heavily on prior thorough knowledge of the system? (Kl)

A. Data driven testing technique

B. Experience-based technique c. White-box technique

D. Structure-based technique

Solution

Again, this is an example of a question where the process of elimination can be implemented. White-box technique and structure-based technique, both of which rely on existing code and design, are synonymous and can be eliminated since there should be only one correct answer. Data driven testing

technique relies on documented test inputs and expected results to run the test. Hence, experience-based technique is the closest answer to the question because knowledge and experience of testers, developers, users, and stakeholders about the system are used to develop test cases.

Answer B is correct.

References

ISTQB (2007a), p. 37 and ISTQB (2007b), p. 13.

117

Sample Exam Questions: ISfQB Certified Tester Foundation Level

21. Which set of test data demonstrates equivalence partitioning

to check whether a customer is a teenager or not? (K3)

A. io. 15, and 19 years

B. 13, 19, and 25 years

C. 13, 16, and 19 years

D. 12, 13, and 20 years

Solution

The age range for a teenager is 13 to 19. With this in mind, the data partitions for a "teenager" test conditions are as follows:

first invalid partition - younger than 13 years old

valid partition - ages between 13 and 19 years old inclusive second invalid partition - older than 19 years old

A review of the response options reveals that only response option D meets the partition criteria listed above.



..

Answer D is correct.

Reference

ISTQB (2007a), p, 38.

118

Sample Exam Questions: ISfQB Certified Tester Foundation Level

22. What technique would be MOST appropriate to check status changes based on certain events? (Kl)

A. state transition

B. Equivalence partitioning

C. Boundary value analysis

D. Decision table

Solution

The phrase "status changes" suggests state transition. Hence, key option A is the correct answer. Equivalence partitioning . technique deals with ranges of values. Boundary value analYSIS complements equivalence partitioning by checking ~~e lower a,nd upper values within a given range. In contrast, decision table IS more suited for enumerating combinations and/or permutations of input or output conditions.

Answer A is correct.

Reference

ISTQB (2007a), p. 38.

119

Sample Exam Questions: ISTQB Certified Tester Foundation Level

Sample Exam Questions: ISTQB Certified Tester Foundation Level

23. USing the diagram below, which test suite will check for ALL valid state transitions using the LEAST effort? (K3)

24. Using the diagram below, which test suite will uncover invalid state transitions for employee status reporting software? (K3)

rt-. \:J

8-

A. so - S1 - S2 - S4 - S1 - S4 - S1 - S2 - S3 - S1

B. 50 - 51 - 52 - 54 - 51 - 52 - 53 - 51

C. 50 - 51 - 54 - 51 - 52 - 53 - 51

D. 50 - 51 - 52 - 54 - 51 - 54 - 51 - 52 - 53

Solution

Take note of two important emphasis words: ALL and LEAST. Response option B did not cover the state transition from 51 to S4. Response option C did not cover the state transition from 52 to 54. Response option D did not cover the state transition from S3 to 51. Only key option A covered all state transitions.

A. Prospective - Active - Resigned - Active - Terminated - Purged

B. Prospective - Active - On Leave - Active - Resigned - Retired

C. Prospective - Active - Retired - Active - On Leave - Purged

D. Prospective - Active - On Leave - Active - Retired - Active

Answer A is correct.

Solution

Reference

ISTQ6 (2007a), p. 38.

Follow the state transition path for each test suite. Response options A, C, and D's test suites aU have valid state transition paths. Response option 6's test suite reveals an invalid state transition, the transition from resigned to retired.

Answer 8 is correct.

120

121

Sample Exam Questions: ISTQB Certified Tester Foundation Level Reference

ISTQB (2007a), p. 39.

122

Sample Exam Questions: ISTQB Certified Tester Foundation Level

25. Which of the following assertions about code coverage are correct? (K2)

A. Statement coverage usually requires more test case suites

B. 100% statement coverage guarantees 100% decision coverage

C. 1000/0 decision coverage implies 1000/0 statement coverage

D. Decision tables cannot be used to list statement coverage values

Solution

Consider the code below.

1: if ((a> b) or (c > d)}

2: e = e + 1;

For statement coverage, only one test case (true values for [a > b] or [c > d]) will be required to achieve 100% execution of all lines of codes (statements 1 and 2). In contrast, decision coverage will require two test cases: statement 1 can either be true or false. Take note that condition coverage will require four test cases as listed below.

a>b c>d Execute Statement 21
True True Yes
True False Yes
False True Yes
False False No Given the example above, response options A and B can be eliminated as valid choices. Decision tables helps document all possible combinations and permutations of tests cases, as shown in the table above.

Answer C is correct.

Reference

ISTQB (2007a), p. 40.

123

temperature speed load
< 0 or> 100 > 100 <- 50
o to 100 N/A N/A Sample Exam Questions: 15TQB Certified Tester Foundation Level

Sample Exam Questions: ISTQB Certified Tester Foundation Level

26. How many test cases are needed to achieve 100% statement coverage? (K3)

if «temperature < 0) Or (temperature> 100» { <;tlert ("DANGER");

~f «speed> 100) and (load <= 50» { speed = 50; .,~

27. Using an error guessing test design technique to convert temperature (Celsius to Fahrenheit, and Fahrenheit to Celsius), experienced testers will MOST UKELY use which set of test data? (K3)

} } else {

check'= false;

A. -1, 0, 89.6, and 212

B. -40,37.78, and 100

C. -1, 0, 1, and 37.78

D. -40, 0, 32, and 100

}

A. 5 8, 4 c. 2 D. 3

Solution

This is an innocent looking K3 type of question. Do not let its simple question format deceive you.

Solution

Error guessing test design technique combines intuition and experience to expose potential failures. Values -1, 0, and 1 seem logical because of boundary values near the freezing point in Celsius. Although most people will not readily know that 212°F is the boiling point of water, most people will know that it is 100°C. However, values 89.6 and 37.78 are arbitrary, and it will be difficult to justify their inclusion as plausible test cases, thus eliminating response options A, 8, and C.

Only two test cases are needed to achieve 100% statement coverage: one test case for when both IF conditions evaluate to be TRUE and another test case for when the first IF condition evaluates to be FALSE (ELSE statement). Keep in mind that the first test case will cover not only the alert statement but also the subsequent IF statement.

Based on experience, one will know that -40°C = ·40°F. Given this, key option D is "more" correct them the other response options.

Answer D is considered correct.

Answer C is correct.

Reference

Reference

ISTQ8 (2007a), p. 41.

ISTQ8 (2007a), p. 40.

124

125

Sample Exam Questions: ISTQB Certified Tester Foundation level

28. By creating future tests based on the results of previous tests, a tester is demonstrating what type of informal test design technique? (Kl)

A. Security testing

B. Non-functional testing

C. Exploratory testing"

D. Interoperability testing

Solution

Exploratory testing is an informal type of technique based on experience that allows a tester to simultaneously learn, design, execute, and log tests. While the system is being tested, the tester learns things that can be used for future test runs.

Interoperability testing assesses the capability of the software product when interacting with another software component or system.

Non-functional testing involves testing software components or a system without reference to functionality.

Security testing checks for vulnerabilities in the system such as unauthorized access or malicious attacks.

Answer C is correct.

Reference

ISTQB (2007a), p. 41.

126

Sample Exam Questions: ISTQB Certified Tester Foundation Level

29. Which of the following are potential drawbacks of independence in testing? (K2)

,

01. Independent testers may feel they are not part of the development team

02. Developers may lose a sense of personal responsibility

for quality

03. Project managers will not have as much control over the

project

04. Customers may end up requesting features that are

technically impossible

A. 01 and 02

B. 01, 02, and 03

C. 03 and 04

D. 01, 02, 03, and 04

Solution

The drawbacks of independent testing are as follows: independent testers may feel isolated from the development

team,

independent testers may impede the software release

process, and

developers may lose a sense of responsibility for quality.







Answer A is correct.

Reference

ISTQB (2007a), p, 45.

127

Sample Exam Questions: ISTQB Certified Tester Foundation Level

30. Which tasks are performed by a test leader versus a tester? (K3)

S. Writing a project test strategy

T. Selecting tools to support testing

U. Preparing and acquiring data V, Scheduling test,?

A. Test leader: 5 and V; Tester: T and U

B. Test leader: S, T, and V; Tester: U

C. Test leader: 5, U, and V; Tester: T

D. Test leader: 5; Tester: T, U, and V

Solution

The tasks of a test leader are as follows:

• coordinate and write the test strategy and plan,

o write test policy, ~

• plan the testing effort, which includes selecting the test approach, estimating the time and cost of testing, selecting and obtaining the proper resources, defining test levels and cycles, and planning incident management,

• initiate the testing effort and control the execution of tests,

• setup configuration management of testware for traceability,

• generate test progress metrics, .

• decide automation details - what needs to be automated, to what degree, and how,

• select tools to support testing and organize appropriate tool

usage training for testers,

• decide on test environment implementation,

• schedule tests, and

• write test summary reports.

128

Sample Exam Questions: lSTQBCertified Tester Foundation Level

The tasks of a tester are as follows:

• 'review and contribute to test plan,

• analyze and review user requirements and specifications for

testability,

• set up test environment,

• prepare and collect test data,

• implement, execute, and log the tests and evaluate and

record the results,

• use required tools (l.e., monitoring tools),

• automate test,

• measure performance of components or system, e review tests developed by others.

Answer B is correct.

Reference

ISTQB (2007a), pp. 45-46.

129

Sample Exam Questions: ISTQB Certified Tester FoUndation Level

31. Which document specifies the sequence of test executions? (Kl)

A. Test procedure specification

B. Test design specification

C. Test case specification

D. Test plan

Solution ~

Test case specification documents the input values, expected results, and executions conditions for a test item.

Test design specification describes the test conditions or coverage items for a test item. Likewise, it specifies the test approach refinements, high level test cases, and pass/fail criteria.

Test plan records the test planning process. It describes the scope, approach, resources, and schedule of testing activities. Also, it identifies the test items (including the pass/fail Criteria), features to be tested, the testing tasks, who will be responsible for each task, and any risks that necessitate contingency planning.

Test procedure specification explains how a tester will run a test. It specifies the sequence of actions or steps to be taken by the tester in order to execute the test. Test procedure specification is also known as test script or manual test script.

Answer A is correct.

References

ISTQB (2007a), p. 47 and ISTQB (2007b), pp, 30-33.

130

Sample Exam Questions: ISTQB Certified Tester Foundation Level

32. Stochastic testing is an example of which test approach or strategy? (K1)

A. Model-based

B. Analytical

C. Methodical

D. Heuristic

Solution

Approach or Strategy Example
Model-based Stochastic testing
Analytical Risk-based testing
Methodical Failure based testil]l
Heuristic Ex_Qloratory testir:!9_ Answer A is correct.

Reference

ISTQB (2007a), p. 48.

131

132

Sample Exam Questions: ISTQB Certified Tester Foundation Level

Sample Exam Questions: ISTQB Certified Tester Foundation Level

33. Which sections are included as part of the test summary report? (K2)

34. Which is a potential product risk factor? (K3)

W. Variances

X. Comprehensive assessment

Y. Evaluation

Z. Summary of activities

A. '. Failure of third-party vendor

B. Training issues

C. Problems requirements definition

D. Poor software functionality

A. W, X, and Y

B. W, X, Y, and Z

C. Wand X

D. W, X, and Z

Solution

Product risk factors are as follows:

Solution

.. delivered software or system is error-prone,

.. software could potentially cause harm to an individual or company,

• poor software characteristics such as functionality, security, reliability, usability, and performance, and

.. software does not perform its intended functions.

A test summary report should include the following:

.. identifier,

..

summary,

Response options A, B, and C are project risk factors.

..

variances,

comprehensive assessment, result summary,

evaluation,

activities or events summary, and approvals .

Answer D is correct.

..

..

Reference

..

..

ISTQB (2007a), p. 52.

Answer B is correct.

References

ISTQB (2007a), p. 49 and IEEE Std 829-1998 (1998), p. 14.

133

Sample Exam Questions: ISTQB Certified Tester Foundation Level

35. Based on the IEEE Standard for Software Test Documentation (IEEE Std 829-1998), in which sections of the test incident report should the following items be recorded? (K2)

Sections

//

a) Test incident report identifier

b) Summary

c) Incident description

d) Impact

Items

1. Impact on test plans

2. Unique identifier

3. Anomalies

4. Procedure step

5. Environment

6. References to other relevant documents

A. a: 2; b: 4; c: 1, 3, and 5; d: 6 8. a: 2; b: 3; c: 4, 5, and 6; d: 1

C. a: 2; b: 6; c: 3, 4, and 5; d: 1

D. a: 2; b: 1; c: 3, 4, and 5; d: 6

Solution

Four sections of the test incident report are listed as a, b, c, and d, and it will be reasonable to assume that they correct. Given this, we just need to map items 1 to 6 to their respective sections.

Item 1 (impact on test plans) clearly belongs to section d (impact), thus ruling out response options A and D as possible ~m:wers. Item ~ (un~que identifier) belongs to section a (test Incident report Identifier). Unfortunately, this is not helpful in eliminating either response option B or C. Both response options Band C have item 4 (procedure step) and item 5 (environment) mapped with section c (incident description).The difference between the two remaining response options is where they map

134

Sample Exam Questions: ISTQB Certified Tester Foundation Level

item 3 (anomalies) and item 6 (reference to other relevant documents). Item 3 clearly belongs to section c, and a ~ummary oflt can potentially be included in section b. However, Item 6 seems out of place in the incident description.

Answer C is correct.

References

ISTQB (2007a), p. 54 and IEEE Std 829-1998 (1998), pp. 13-14.

135

Sample Exam Questions: ISTQB Certified Tester Foundation Level

36. Based on the IEEE Standard for Software Test Documentation (IEEE Std 829-1998), which of the following sections are part of the test summary report? (K1)

a) Test summary and report identifier

b) Comprehensive assessment

c) Summary of results

d) Evaluation

e) Observers

f) Approvals

A. a, b, c, d, and e

B. a, b, c, e, and f

C. a, c, o, e, and f

D. a, b, c, d, and f

Solution

The test summary report includes the following section:

e

identifier, summary, variances,

comprehensive assessment, results summary, evaluation,

activities or events summary, and approvals .

..

..

..





Observers is part of the incident description section of the test incident report. It is not even a section, hence eliminating response options A, B, and C.

Answer D is correct.

References

ISTQB (2007a), p. 54 and IEEE Std 829-1998 (1998), p. 14.

136

Sample Exam Questions; ISTQB Certified Tester Foundation Level

37. What is the name of a temporary software component that is used to call another component for testing purposes? (K1)

A. Domain

B. Use case

C. Stub

D. Driver

solution

Domain and use case can easily be eliminated because they are not software components. A stub replaces a "called" program, while a driver replaces a "calling" program.

Answer D is correct.

References

ISTQB (2007a), p. 57 and ISTQB (2007b), pp. 15 and 29.

137

Sample Exam Questions: ISTQB Certified Tester Foundation Level

Sample Exam Questions: ISTQB Certified Tester Foundation Level

38. Which of the following is a potential risk in using test support tools? (Kl)

39. How are (a) static analysis tools and (b) performance testing tools different? (K2)

A. (a) helps in enforcing coding standards; (b) tests system performance

6. (a) analyzes security vulnerabilities; (b) measures the effectiveness of test cases

C. (a) prepares codes prior to testinq; (b) prepares codes prior to stress testing

D. (a) highlights unreachable conditions; (b) improves system performance

A. Underestimating the effort needed to maintain the test assets

6. Losing access to important testing information when needed

C. Relying too much on qualitative and quantitative assessments

D. Lowering the morale of the test team because of repetition

Solution

The risks of using test support tools are as follows:

Solution



unrealistic expectations of tool performance which may include functionality and ease of use,

underestimate time, cost, and effort to introduce and train users in usage of the tool,

underestimate time and effort needed to gain significant benefits from tool usage,

underestimate effort involved in maintaining the test resources or assets produced by the tool, and overdependence on the tool,

Performance testing tools help simulate increased load, volume, and stress on the system to monitor and report the behavior of the system under intense conditions.





Static analysis tools help developers, testers, and QA staff enforce coding standards, analyze structures, verify dependencies, and understand the code before performing dynamic testing.



Answer A is correct.

Answer A is correct.

Reference

Reference

ISTQ6 (2007a), pp. 60-62.

ISTQB (2007a), p, 62.

138

139

Sample Exam Questions: ISTQB Certified Tester Foundation level

40. Which of the following is a potential pilot project objective when introducing a test support tool into an organization? (Kl)

A. Measuring the satisfaction of management for staying within scope

B. Assessing whethetthe benefits will be achieved at reasonable cost

C. Receiving compliments from the users on the aesthetic aspects of the tool

D. Reducing the amount of overtime need to finish the project on time

Solution

Potential objectives of a pilot project are as follows:

• learn and discern more details about the toot,

• assess adequacy of the tool with existing processes and practices; does it fit with the current process or should changes be made?,

• decide on procedures of using, managing, storing, and maintaining the tool and the test resources,

• perform benefit and cost analysis; will the benefits of using the tool be achieved at a reasonable cost?

Answer B is correct.

Reference

ISTQB (2007a), p. 64.

140

Chapter 6: PrC!.~~~"~~,9~~~!"i~~~,~_~~,,""",._._ .... _

m .. '.~ ... ' .. 'm~_~._ ... w.w_w •• _.w~· .. ·_. ._._. __

1. 'Which approaches can help increase the quality of software?

I. Incorporating rigorous testing II. Preventing change requests III. Establishing defects metrics

IV. Allocating schedule contingencies

A. I and II are true; III and IV are false

6. III and IV are true; I and II are false

C. I and IV are true; II and III are false

D. I and III are true; II and IV are false

2. Which of the following can help testers understand the root causes of defects from previous projects?

A. Ishikawa diagram

B. Cause-and-effect diagram

C. Lessons learned

D. Fishbone diagram

3. What would USUALLY have a set of input values and execution conditions?

A. Test basis

B. Test case

C. Test objective

D. Test control

141

Sample Exam Questions: ISTQS Certified Tester Foundation Level

4. Which of the foHowing general testing principles are true?

1. Testing shows the presence of defects but not the absence of defects

II. Testing of combinations of inputs and outputs will find aU defects

III. Testing should start after the completion of key development tasks

IV. Testing of safety-critical software is similar to testing web applications

A. I is true; II, III and IV are false

B. II is true; If III and IV are false

C. I and II are true; III and IV are false

D. II and III are true; I and IV are false

5. Which of the following is a MAJOR test planning task?

A. Determining the exit criteria

B. Measuring and analyzing results

C. Implementing corrective actions

D. Monitoring and documenting progress

6. Which of the following is a KEY test closure task?

A. Ensuring proper environment setup

B. Writing a test summary report

C. Assessing the need for additional tests

D. Finalizing and archiving testware

142

Sample Exam Questions: ISTQS Certified Tester Foundation Level

7. Which statements BEST demonstrate the various degrees of independent testing?

A. Tests are designed using exhaustive test design techniques Tests are designed using exploratory test design techniques

B. Tests are designed to generate most fundamental errors early

Tests are designed to uncover errors with the highest risk

C. Tests are designed by the person who wrote the code Tests are designed by a person from another department

D. Tests are designed based on the documented requirements Tests are designed based on the experience of the testers

8. Which of the following software work product can be used as a basis for testing?

A. Incremental scenarios

B. Design documents

C. Undocumented features

D. V-model specifications

9. Which tests are BEST described by the following characteristics?

W. Component testing

X. Integration testing

Y. Alpha testing

Z. Robustness testing

L Testing the interaction between components

2. Fixing defects as soon as they are found

3. Automating test cases before coding

4. Testing separately testable components

A. Wl, X4, Y3 and Z2

B. W2, W4, Xl and Zl

C. W2, W3, W4 and xi

D. W3, xi, X2 and X4

143

144

Sample Exam Questions: ISTQB Certified Tester Foundation Level

Sample Exam Questions: ISTQB Certified Tester Foundation Level

10. Which test is OFTEN the responsibility of the customers or users of the system?

1S. As a moderator in a typical formal review, what can be one , of your responsibilities?

A. Usability testing

B. Functional testing

c. Maintenance testing D. Acceptance testing c.

11. Which acceptance test is USUALLY performed by system

administrators? '

A. Deciding on the execution of reviews

B. Documenting all the issues and problems

C. Leading the review of the documents

D. Identifying and describing the findings

A. Operational

B. Customer

C. Contractual

D. Regulatory

16. What is the KEY difference in the usage of static analysis tools?

I. Functional testing

II. Non-functional testing III. Structural testing

A. Developers use static analysis tools before and during component testing

Designers use static analysis tools during software modeling

B. Developers use static analysis tools to check the syntax of their codes

Designers use static analysis tools to ensure adherence to programming standards

C. Developers use static analysis tools before and after integration testing

Designers use static analysis tools to guarantee regulatory compliance

D. Developers use static analysis tools to check the syntax of their codes

Designers use static analysis tools after software modeling

12. Which test can be performed at all test levels?

A. System testing

B, Operational testing

C. Structural testing

D. Integration testing

13. RegreSSion testing can be applied to which of the following?

A. I and II only

B. I and III only

C. II and III only D, I, II and III

17. Features to be tested, approach refinements and feature pass/fail criteria BUT excluding environmental needs should be speofied in which document?

14. Which test requires the execution of the software component?

A. Test case specification

B. Test plan

C. Test procedure specification

D. Test design specification

A. Formal inspection

B. Dynamic testing

C. Code walkthrough

D. Execution testing

145

Sample Exam Questions: ISTQB Certified Tester Foundation Level

18. Which document specifies the execution order of test cases?

A. Test design specification S. Test item

C. Test procedure specification

D. Test plan

19. Match the test design techniques to the correct descriptions.

S. Black-box technique

T. White-box technique

U. Structural-based technique

V. Specification-based technique

1. Selecting test cases based on documentation

2. Ignoring the internal structure of the system

A. 51,52, Ul and U2

B. Tl, T2, Ul and U2

C. 51, 52, Vi and V2

D. Til T2, Vi and V2

20. What techniques would be MOST appropriate if the specifications are outdated?

A. Structure-based and experienced-based techniques

B. Black-box and specification-based techniques

C. Specification-based and structure-based techniques

D. Structure-based technique and exhaustive testing

21. To test an input field that accepts a two-digit day based on a particular month which data set demonstrates boundary value analysis?

A. 0, 1, 16, 31 and 100 S. l,2~2~30and31

C. 21 26, 27, 29 and 30

D. -1, 0, 15, 32 and 99

146

\

to • ISTQB Certified Tester Foundation Level

sample Exam QUes IOns.

. hni lf OffEN considered as an extension of

22. Which tee mque I

equivalence partitioning?

A. Decision table testing

B. State transition testing

C. Use case testing

D. Boundary value analysis

23. Without testing all possible transitions, which test suite will test all marital statuses?

A. SO - 51 - 52 - 54 - 51 - ~~

B. 50 - 51 - S2 - 53 - 51 - 53 50 - 51 - S4 - 51 - 52 -

D. SO - 51 - 52 - 53 - S4 - 51

24. Which technique describes process flows through a system based on its likely usage?

A. Data driven testing

B. State transition testing

C. Decision table testing

D. Use case testing

147

Sample Exam Questions; ISTQB Certified Tester Foundation level 25, Which combination of p, q and r values will ensure 100% statement coverage?

if (p "" q) {

r "" r + 1; if (r < 5) { S ". 10;

} } else if (p >~q) {

}

s "" 5;

A. p::;::: 5, q ::;::: 5, r =:: 5 p ::;::: 5, q ::;::: 4, r ::::: ~ 1

B. p::::: 5, q :::: 1, r :::: 3 p ::: 4, q ::: 4, r ;;:: 5

C. P "" 3, q ::;::: 3, r ::: 3 p ::;::: ~ 1, q :::: -2, r ::::: 3

D. p:::: ~ 1, q = ~ 1, r = 0 p :::: ~2, q ::::: ~1, r ::::: 0

26. How many test cases are needed to achieve 100% condition coverage?

if «temperature < 0) or (temperature> 100») { ~lert ("DANGER");

~f «speed> 100) and (load <= 50)} { speed = 50;

} } else {

check"" false;

}

A, 5

B. 4

C. 2

D. 3

148

Sample Exam Questions: ISTQB Certified Tester Foundation Level 27, What is an informal test design technique where a tester uses information gained while testing to design new and better tests?

A. Error guessing

B, Exploratory testing

C. Use case testing

D. Decision table testing

28, How are error guessing and exploratory testing similar?

A, Both are widely used formal techniques

B, Both are white-box test design techniques

C. Both are experience-based testing

D. Both are effective at aU testing levels

29. Who should be responsible for coordinating the test strategy with the project manager and others?

A. Tester

B. Developer

C. Customer

D. Test leader

3D. Which of the following are KEY tasks of a test leader?

i. Understanding the project risks

ii. Measuring performance of components iii. Scheduling tests and other activities

iv. Using monitoring tools as needed

A. i and iii

B. i and ii

C. iii and iv

D. ii and iii

149

150

Sample Exam Questions: ISTQB Certified Tester Foundation Level

34. What are the main objectives of software project risk management?

A. Increase focus on preventive processes and improve tester job satisfaction

B. Reduce the probability of occurrence and decrease the

potential impact

C. Control contractor problems and minimize the impact of

corporate politics

D. Increase the probability of project success regardless of the

cost involved

Sample Exam Questions: ISTQB Certified Tester Foundation Level

31. What is a group of test activities that are organized and managed together?

A. Test procedure specification

B. Test level

C. Test case specification

D. Test plan

32. Which test approaches or strategies are characterized by the

descriptions below? ~

s. Process-compliant approaches

T. Heuristic approaches

U. Consultative approaches

V. Regression-averse approaches

35. Based on the IEEE Standard for Software Test Documentation (IEEE 5td 829-1998), which sections of the test incident report should the following details be recorded?

1. Includes reuse of existing test material

2. Listens to suggestions from technology experts

3. Adheres to industry-specific standards

4. Runs test execution and evaluation concurrently

a ) Test incident report identifier

b) Summary

c) Incident description

d) Impact

1. Expected results 2, Actual results

3, procedure step 4, Environment

5. Revision level

6. Date and time

A. 54, T3, U2, V1

B. si. T2, U3, V4 C, 52, T3, U1, V4 D, 53, T4, U2, V1

33. What is the ratio of the number of failures relative to a

category and a unit of measure? .

A. Failure rate

B, Defect density

C. Failure mode

D. Fault tolerance

A. a: 3; b: 5; c: 1, 2, 4 and 6

B. b: 5; c: 1, 2, 3, 4 and 6

C. b: 5 and 6; c: 1, 2, 3 and 4

D. a: 5; c: 1, 2, 3, 4 and 6

151

Sample Exam Questions: ISTQB Certified Tester Foundation Level

Sample Exam Questions: ISTQB Certified Tester Foundation Level

39. Which tool needs to interface with other office automation software in order to generate reports in the format required . by the organization?

36. Based on the IEEE Standard for Software Test Documentation (IEEE Std 829-1998), which of the following sections are part of the test summary report?

a) Summary

b) Test incident report identifier

c) Test deliverables

d) Risks and contingencies

e) Variances

f) Approvals

g) Output specifications

A. Progress tracking tools

B. Test management tools

C. Metrics management tools

D. Test execution tools

40. Which of the following can be considered as success factors

when deploying a new tool in an organization?

A. Providing coaching to users and defining usage guidelines .

B. Monitoring tool usage and reducing the need for risk analysis C, Improving processes and focusing more on component

testing .

D. Assessing testing completion and minimizing code reviews

A. a, e and f

B. a, c and d

C. a, band f

D. a, d and e

37, Which tool can be used to support and control part of the test management process?

A. Coverage management tool

B. Test management tool

C. Data preparation tool

D. Performance testing tool

38. What is a scripting technique that uses data files to contain not only test data and expected results, but also keywords related to the application being tested?

A. Automation technique

B. Scripting language

C. Process-driven testing

D. Keyword-driven testing

152

153

Sample Exam Questions: ISTQB Certified Tester Foundation Level Correct Answers and Cognitive Levels

1 D K2
2 C Kl
3 B K1
4 A K2
5 A K1
6 0 K1
7 C K2
8 B K1
9 C K2
10 0 Kl
11 A Kl
12 C K1
13 0 K2
14 B K1
15 C Kl
16 A K2
17 0 K3
18 C Kl
19 C K2
20 A K3 21 B K3
22 0 K1
23 C K3
24 0 K1
25 C K3
26 A K3
27 B K1
28 C K2
29 0 K1
30 A K2
31 B K1
32 D K2
33 A K3
34 B K2
35 B Kl
36 A K3
37 B Kl
38 0 Kl
39 B Kl
40 A K2 154

1. Which approaches can help increase the quality of software? (K2)

I. Incorporating rigorous testing II. Preventing change requests III. Establishing defects metrics

IV. Allocating schedule contingencies

A. I and II are true: III and IV are false

B. III and IV are true; I and II are false

C. I and IV are true; II and III are false

D. I and III are true; II and IV are false

Solution

The quality of software can be measured by the number of defects found for both functional and non~functional requirements and characteristics. Through the process of testing, defects are identified, investigated (includes change request), and corrected. With this in mind, approaches I and III are valid methods of increasing software quality.

Answer D is correct.

Reference

ISTQB (2007a), p. 11.

155

156

Sample Exam Questions: ISTQB Certified Tester Foundation Level

Sample Exam Questions: ISTQB Certified Tester Foundation level

2. Which of the following can help testers understand the root

causes of defects from previous projects?·(Kl)

A. Ishikawa diagram

B. cause-eno-enect diagram

C. Lessonslearned

D. Fishbone diagram

3. What would USUALLY have a set of input values and execution conditions? (K1)

A. Test basis

B. Testcase

C. Test objective

D. Test control

Solution

Solution

Ishikawa diagram (also known as cause-and-effect diagram or fish bone diagram) helps identity potential causes of a problem. However, because there is one and only one correct answer per question, response options A, B, and D can be ruled out as the correct answer. Moreover, the phrase "from previous projects" hints at something that happened in the past. Thus, key option C is the correct answer.

Test basis is defined as all documents from which the requirements of a system can be gathered or surmised. It is the document from which test cases are based.

Reference

Test case is defined as a set of test data (or input values), test scripts (or execution conditions), and expected results developed for the purpose of verifying one or more system requirements.

Test control is a test management task that deals with applying corrective actions to set the test project on track.

Test objective states the reason or purpose for designing and executing a test.

Answer C is correct.

ISTQB (2007a), p. 12.

Answer B is correct.

Reference

ISTQB (2007a), p. 13 and ISTQB (2007b), pp. 30 and 32.

157

Sample Exam Questions: ISTQB Certified Tester Foundation Level

4. Which of the following general testing principles are true? (K2)

1. Testing shows the presence of defects but not the absence of defects

II. Testing of combinations of inputs and outputs will find all defects

III. Testing should start after the completion of key development tasks

IV. Testing of safety-critical software is similar to testing web applications

A. I is true; II, III, and IV are false

B. II is true; I, III, and IV are false

C. I and II are true; III and IV are false

D. II and III are true; I and IV are false

Solution

The seven major principles of testing are as follows:

..

testing shows the presence but not absence of defects, exhaustive testing will not yield a defect-free system, testing should commence as early as possible in the system development life cycle,

a smaU number of modules contain most of the defects found during pre-release testing, or show the most operational failures,

overcome the pesticide paradox-if the same tests are performed frequently using the same test cases, eventually no defects will be discovered,

testing is done differently for every different context, and absence of errors does not guarantee that the system is usable and users' requirements are met and fulfilled.



..



..

..

..

Answer A is correct.

Reference

ISTQB (2007a), p. 14.

158

Sample Exam Questions: ISTQB Certified Tester Foundation Level

S. Which of the following is a MAJOR test planning task? (K1)

A. Determining the exit criteria

B. Measuring and analyzing results

C. Implementing corrective actions

D, Monitoring and documenting progress

Solution

The major tasks of test planning are as follows:

• determine and identify the scope, risks, and objectives of

testing!

.. determine the test approach,

.. determine the required test resources,

.. implement the test policy and/or test strategy,

• schedule test analysis and design tasks,

.. schedule test implementation, execution 1 and evaluation,

and

• determine the exit criteria.

Response options 8, C, and D are major test control tasks.

Answer A is correct •

Reference

ISTQB (2007a), p. 15.

159

Sample Exam Questions: ISTQB Certified Tester Foundation Level

6. Which of the following is a KEY test closure task? (K1)

A. Ensuring proper environment setup

B. Writing a test summary report

C. Assessing the need for additional tests

D. Finalizing and archiving testware

Solution

The major tasks of the test closure activity are as follows:

..

check which planned deliverables have been delivered, verify closure of incident reports, and document acceptance of the system,

finalize and archive testware, the test environment, and infrastructure for future reference and reuse,

entrust testware to the maintenance organization, and analyze lessons learned for future projects .

..



..

Response option A is a major task of the test implementation activity, while response options Band C are major tasks of the evaluating exit criteria activity,

Answer D is correct.

Reference

ISTQB (2007a), p. 16.

160

Sample Exam Questions: ISTQB Certified Tester Foundation Level

7. Which statements BEST demonstrate the various degrees of independent testing? (K2)

A. Tests are designed using exhaustive test design techniques Tests are designed using exploratory test design techniques

B. Tests are designed to generate most fundamental errors early

Tests are designed to uncover errors with the highest risk

C. Tests are designed by the person who wrote the code Tests are designed by a person from another department

D. Tests are desiqned based on the documented requirements Tests are designed based on the experience of the testers

Solution

The primary purpose of independent testing is to eliminate author bias in finding defects. Hence, test design and creation are done by a person or people who mayor may not be familiar with the system.

Response options A, B, and 0 do not identify who is designing the test. Rather, they state what technique or approach is being used to define the test.

Answer C is correct.

Reference

ISTQB (2007a), p. 17.

161

Sample Exam Questions: ISTQB Certified Tester Foundation Level

8. Which of the following software work products can be used as a basis for testing? (K1)

A. Incremental scenarios

B. Design documents

C. Undocumented features

D. V-model specificatiof1,~

Solution

Software work pr.odUcts are deliverables (documents, artifacts, etc.). suc~ as business case, requirements document, and design s~lfic~tlon. Only key option B is a software work product, thus eliminating response options A, C, and D.

Answer B is correct.

Reference

rSTQB (2007a), p. 20.

162

Sample Exam Questions: ISTQB Certified Tester Foundation Level

g. Which tests are BEST described by the following characteristics? (K2)

W. Component testing

X. Integration testing

Y. Alpha testing

Z. Robustness testing

1. Testing the interaction between components

2. Fixing defects as soon as they are found

3. Automating test cases before coding

4. Testing separately testable components

A. Wl, X4, Y3, and Z2

B. W2, W4, Xl, and Zl

C. W2, W3, W4, and xi

D. W3, Xl, X2, and X4

Solution

Component testing involves testing an individual hardware or software unit, component, or module.

Integration testing exposes defects in the interfaces and interactions between components.

.

Alpha testing is defined as simulated or actual operational testing by potential users/customers or an independent test team at the developers' site, but outside the development organization.

Robustness testing determines how a system operates correctly in the presence of invalid inputs.

Given the definitions above, we can easily eliminate response options A and D, both of which describe integration testing using the characteristic of a component testing-testing separately testable components. Response option B correctly describes component testing and integration testing; howev~r, robu~tness testing is described using the characteristics of an integratIOn testing.

163

Sample Exam Questions: ISTQB Certified Tester Foundation Level

Answer C is correct.

References

ISTQB (2007a), p. 22 and ISTQB (2007b), pp. 7 and 26.

164

Sample Exam Questions: ISTQB Certified Tester Foundation Level

10. Which test is OFTEN the responsibility of the customers or users of the system? (K1)

A. Usability testing

B. Functional testing

C. Maintenance testing

D. Acceptance testing

Solution

The emphasis word OFTEN hints at the correct answer to this question. Although all response options listed above can be performed by the development team, it is the user or the customer who generally performs the acceptance test of a system. Acceptance testing establishes confidence among users that the system is ready for use.

Answer D is correct.

Reference

ISTQB (2007a), p. 23.

165

Sample Exam Questions: ISTQB Certified Tester Foundation Level

Sample Exam Questions: ISTQB Certified Tester Foundation level 12. Which test can be performed at all test levels? (Kl)

11. Which acceptance test is USUALLY performed by system administrators? (K1)

A. Operational

B. Customer

C. Contractual

D. Regulatory

A. System testing

B. Operational testing

C. Structural testing

D. Integration testing

Solution

Solution

The process of elimination can help narrow down the a~swer for this question. Response options A and D are test levels In and of themselves and therefore the respective tests can be performed only at the specified level. Response option B is a type of acceptance test and can be performed only during acceptance testing. Hence, response option C is the reasonable answer.

All but response option A are performed by users. Operational acceptance tests are done by the system administrator. This type of acceptance test usually includes the test of ba~kup/restore procedure, disaster recovery, user management, maintenance tasks, and periodic check of security vulnerabilities.

Answer A is correct.

166

Answer C is correct.

Reference

Reference

ISTQB (2007a), p. 24.

ISTQB (2007a), p. 25.

167

Sample Exam Questions: ISTQB Certified Tester Foundation Level

13. Regression testing can be applied to which of the following? (K2)

1. Functional testing

II. Non-functional testing III. Structural testing

A. I and II only

B. I and III only c. II and III only D. I, II, and III

Solution

Regression testing ensures that defects were not introduced to a previously tested system after it was modified or changed. It can be performed at all test levels and applies to functional, nonfunctional, and structural testing.

Answer D is correct.

Reference

ISTQB (2007a), pp. 25-26.

168

Sample Exam Questions: ISTQB Certified Tester Foundation Level

14. Which test requires the execution of the software component? (K1)

A. Formal inspection

B. Dynamic testing

C. Code walkthrough

D. Execution testing

Solution

Using the process of elimination, answers to this question could be narrowed down. Response options A and C are both manual reviews and are static testing techniques, hence eliminating them as possible answers. Response option D is a plausible distracter, but and nowhere in the syllabus or glossary is the term "execution testing" ever mentioned. Therefore, the only feasible correct answer is response option B.

Answer B is correct.

Reference

ISTQB (2007a), p. 29.

169

Sample Exam Questions: ISTQB Certified Tester Foundation Level

15. As a moderator in a typical formal review, what can be one

of your responsibilities? (Kl)

A. Deciding on the execution of reviews

B. Documenting all the issues and problems

C. Leading the review of the documents

D. Identifying and describing the findings

Solution

Sample Exam Questions: ISTQB Certified Tester Foundation Level

16. What is the KEY difference in the usage of static analysis tools? (K2)

A. Developers use static analysis tools before and during component testing

Designers use static analysis tools during software modeling

B. Developers use static analysis tools to check the syntax of their codes

Designers use static analysis tools to ensure adherence to programming standards

C. Developers use static analysis tools before and after integration testing

Designers use static analysis tools to guarantee regulatory compliance

D. Developers use static analysis tools to check the syntax of their codes

Designers use static analysis tools after software modeling

The responsibilities of a moderator in a formal review are as foHows:



lead the review of the documents or set of documents plan the review meeting, I

run the review meeting,

follow up after the review meeting, and

mediate, if necessary, between different points of view.





. Solution

Developers use static analysis tools to check for code standards and syntax compliance before and during component and integration testing. Designers also use sta~ic analysis tool to . check for design standard compliance dunng software modeling,

Response option B is incorrect because it is the developer and not the designer who uses the tool to ensure adherence to programming standards, Response options C and D ,are both . incorrect as well because of the time when the tool IS used-It should be during and not after integration testing and software modeling, respectively,

Response options A, B, and D are the responsibility of a manager, an author, and reviewers, respectively.

Answer C is correct.

Reference

ISTQB (2007a), p, 30.

Answer A is correct.

Reference

ISTQB (2007a), p. 33.

170

171

Sample Exam Questions; ISTQB Certified Tester Foundation Level

17. Features to be tested, approach refinements, and feature pass/fail criteria BUT not environmental needs should be specified in which document? CK3)

A. Test case speciflcatlon

B. Test plan

C. Test procedure specification

D. Test design specification

Solution

Test case speciflcation identifies the input values, expected results, and executions conditions for a test item.

Test design specification describes the test conditions or coverage items for a test item or feature. Likewise, it specifies the test approach refinements, high level test cases, and pass/fail criteria.

Test plan records the test planning process. It describes the scope, approach, resources, and schedule of testing activities. Also, it identifies the testing tasks, who will be responsible for each task, and any risks that necessitate contingency planning.

Test procedure specification explains how a tester will run a test. It specifies the sequence of actions or steps to be taken by the tester in order to execute the test. Test procedure specification is also known as test script or manual test script,

Answer D is correct.

References

ISTQB (2007a), p. 36 and ISTQB (2007b), pp. 30-33.

172

Sample Exam Questions: ISTQB Certified Tester Foundation Level

18. Which document specifies the execution order of test cases? (Kl)

A. Test design specification

B. Test item

c. Test procedure specification

D. Test plan

Solution

Test design spedticatlon documents the test conditions or coverage items for a test item or feature, test approach refinements, high level test cases, and pass/fail criteria.

Test item is defined as the specific component or feature to be tested.

Test plan records the test planning process. It describes the scope, approach, resources, and schedule of testing activities.

Test procedure specification explains how a tester will run a test. It specifies the sequence of actions or steps to be taken by the tester in order to execute the test.

Answer C is correct.

References

ISTQB (2007a), p. 36 and ISTQB (2007b), pp. 30-33,

173

Sample Exam Questions: ISTQB Certified Tester Foundation Level

Sample Exam Questions: ISTQB certified Tester Foundation Level

20. What techniques would be MOST appropriate if the

specifications are outdated? (K3)

A. Structure-based and experienced-based techniques

B. Black-box and specficatlon-based techniques

c. Specification~based and structure-based techniques D. Structure-based technique and exhaustive testing

19. Match the test desiqn techniques to the correct descriptions. (K2)

S. Black-box technique

T. White~box technique

U. Structural-based technique

v. Specification-based techRique

1. Selecting test cases based on documentation

2. Ignoring the internal structure of the System

A. 51, 52, U1, and U2

B. ri. T2, U1, and U2

C. S1, 52, V1, and V2

D. ri, T2, VI, and V2

Solution

Solution

Again, this is an example of a question where th~ process of elimination can be implemented. Black-box techmque and

spedttcation-based technique both rely on the analysi~ of. .

existing specifications to derive test cases. If the specification IS outdated it is likely that invalid test cases will be produced or developed. For this reason, we can eliminate response options B and C. Response option D is a plausible answer except that exhaustive testing is not a technique but a test approach. Response option A is considered to be the correct answer.

BJack~box technique and white-box technique are synonymous with specification-based technique and structural-based technique, respectively. This means that the characteristics of the former technique wilt be the same as for the latter.

Answer A is correct.

Based on the explanation given above, the test design technique pairing shou Id be 5 with V and T with U, thus eliminating response options A and D. Response option B has the correct pairing of test design technique; however, description 2 contradicts the definition and characteristic of both white-box and structural~based test design technique.

Reference

Answer C is Correct.

ISTQB (2007a), p. 37.

Reference

ISTQB (2007a), p. 37.

174

175

Sample Exam Questions: ISTQB Certified Tester Foundation Level

21. To test an input field that accepts a two-digit day based on a particular month, which data set demonstrates boundary value analysis? (K3)

A. 0, 1, 16, 31, and 100

B. 1,27,28,30, and 31

C. 2, 26, 27, 29, and 30

D. -1, 0, 15, 32, and 99

Solution

The lower boundary value for all months is 1. For February, the upper boundary value can be either 28 or 29. The rest of the months have either 30 or 31 days.

The value of 100 is a good test case for response option A because it checks for more than two digits. However, the value of 16 serves no purpose from a boundary value perspective.

In response option D, values 0 and 32 are valid test cases because they are immediately adjacent to the lower (1) and upper (31) boundary values respectively. However, values -1 and 99 are not immediately adjacent to the lower and upper boundary values.

Response options Band C are both acceptable answers. However, key option B is superior to response option C because it includes the upper boundary value of 31.

Answer B is considered correct.

Reference

ISTQB (2007a), p. 38.

176

Sample Exam Questions: ISTQB Certified Tester Foundation level

22. Which technique is OFTEN considered as an extension of equivalence partitioning? (K1)

A. Decision table testing

B. State transition testing

C. Use case testing

D. Boundary value analysis

Solution

Boundary value analysis complements equivalence partitioning by checking the lower and upper values within a given range.

Decision table testing identifies conditions and actions of the system. Such conditions will be documented in a table to depict various combinations of conditions and the associated actions.

State transition testing focuses on how states (status or condition) transition from one status to the next depending on

events and history.

Use case testing checks the interaction between business or user scenarios.

Answer D is correct.

Reference

ISTQB (2007a), p. 38.

177

Sample Exam Questions: ISTQB Certified Tester Foundation Level

23. Without te~ing all possible transitions, which test suite will test all mantal statuses? (K3)

A. 50 - 51 - 52 - 54 - 51 - 54

B. 50 - 51 - 52 - 53 - 51 - 52

C. 50-51-54-51-52_ 53

D. 50 - S1 - S2 - 53 - 54 - 51

Solution

Trn: word "all" in this question hints at the answer. Response options A and B clearly can be eliminated as possible answers because they do not cover or test for 53 and S4 respectively The state-tra~sitio~ from S3 to 54 does not eXist, thus makin~ response option D Incorrect. Only key option C covers all marital statuses and has valid state transitions.

Answer C is correct.

Reference

ISTQB (2007a), p. 38.

178

Sample Exam Questions: ISTQB Certified Tester Foundation Level

24. Which technique describes process flows through a system based on its likely usage? (Kl)

A. Data driven testing

B. 5tate transition testing

C. Decision table testing

D. Use case testing

Solution

Data driven testing is a scripting technique which generally supports execution of capture or playback tools.

Decision table testing identifies conditions and actions of the system. 5uch conditions will be documented in a ,table to ?epict various combinations of conditions and the associated actions.

5tate transition testing focuses on how states (status or condition) transition from one status to the next depending on events and history.

Use case testing checks the interaction between business or user scenarios. It is most useful in describing and uncovering defects in the process flow during real-time use of the system.

Answer 0 is correct.

Reference

rSTQB (2007a), p. 39.

179

Sample Exam Questions: ISTQB Certified Tester Foundation Level

25. Which combination of p, q, and r values will ensure 100% statement coverage? (K3)

if (p = q) {
r = r + 1-
,
if (r < 5) {
s ... lOi
}
} else if (p > q) {
s = 5 i."
} A. P = 5, q ::::: 5, r = 5 P = 5, q == 4, r == -1

B. p= 5, q ::::: 1, r :::: 3 p = 4, q = 4, r = 5

C. P = 3, q = 3, r = 3 P = -1, q = -2, r = 3

D. P = -1, q = -1, r = 0 p = -2, q = -1, r ::::: 0

Solution

In the question above, two test cases will ensure 100% statement coverage (all statements or lines of codes will be executed).

Test case 1: p== q and r < 5 Test case 2: p > q

Response options A and B can be eliminated because both have r = 5, consequently not satisfying test case 1. Response options C and D are plausible answers. However, the second combination of p and q values (p ::::: -2 and q = -1) in response option 0 does not satisfy test case 2. Note that -1 is greater than -2. Only key option C covers aU the test cases, thus ensuring 100% statement coverage.

Answer C is correct.

Reference

ISTQB (2007a), p. 40.

180

Sample Exam Questions: ISTQB Certified Tester Foundation Level

26. How many test cases are needed to achieve 100% condition coverage? (K3)

if ({temperature < 0) or (temperature> 100» { alert ("DANGER");

if «speed> 100) and (load <= 50» {

speed = 50i

}

} else {

check = false;

}

A. 5 S. 4

C. 2

D. 3

Solution

The following test cases will ensure 100% condition coverage:

Test case 1: (temperature < 0 or temperature> 100) and (speed> 100 and load <= 50)

Test case 2: (temperature < a or temperature> 100) and (speed> 100 and load> 50)

Test case 3: (temperature < 0 or temperature> 100) and (speed <= 100 and load <== 50)

Test case 4: (temperature < a or temperature> 100) and

(speed <== 100 and load> 50)

Test case 5: (temperature >== 0 and temperature <= 100)

Keep in mind that the first IF statement will be covered as long as one of the conditions returns a value of TRUE.

Answer A is correct.

Reference

ISTQB (2007a), p. 40.

181

Sample Exam Questions: ISTQa Certified Tester Foundation Level

27. What is an informal test design technique where a tester uses information gained while testing to design new and better tests? (Kl)

Sample Exam Questions: ISTQB Certified Tester Foundation Level

28, How are error guessing and exploratory testing similar? (K2)

A. Error guessing

B. Exploratory testing

C. Use case testing "

D. Decision table testing

A. Both are widely used formal techniques

B. Both are whlte-box test design techniques

C. Both are experience-based testing D, Both are effective at all testing levels

The key words "while testing" hints at the answer to this question. Only exploratory testing, an experience.based technique, allows a tester to concurrently perform test execution and test desiqn, The other techniques let testers create test d~signs via s~dfications, models, code structure, or experience Without the aid of test execution.

solution

The process of eHmination can help narrow down. the answ.er for this question. Error guessing and exploratory testing are Widely used techniques that rely on the tester's skills, .knowl~ge, an.d system familiarity to create or develop test desiqns, G~ven this concept, we can eliminate response options A and 6, since ~oth techniques are not formal and not wnite-box, Response option D provides a good analogy but is a less appropri~te answ~r because of the word "all." Thus, key option C IS a supenor choice.

Solution

Answer B is correct.

Answer C is considered correct.

Reference

ISTQB (2001a), p. 41.

Reference

ISTQB (2007a), p. 41.

182

183

Sample Exam Questions: ISTQB Certified Tester Foundation Level

29. Who should be responsible for coordinating the test strategy with the project manager and others? (K1)

A. Tester

B. Developer

C. Customer

D. Test leader

Solution

One major task of a test leader is to coordinate and plan the test strategy with project managers and other people (including but not limited to developers, testers, and users) involved in the testing effort. Developers, testers, and/or users may contribute to the formulation of the test plan.

Answer D is correct.

Reference

ISTQB (2007a), p, 45.

184

sample Exam Questions: ISTQB Certified Tester Foundation Level

30. Which of the following are KEY tasks of a test leader? (K2)

l, Understanding the project risks

ii, Measuring performance of components iii. Scheduling tests and other activities

iv, Using monitoring tools as needed

A. i and iii

B. i and ii

C. iii and iv

D. ii and iii

Solution

The tasks of a test leader are as follows:



coordinate and write the test strategy and plan,

write test policy, .

plan the testing effort which includes selecting, the test , approach, estimating the time and cost of testing, selecting and obtaining the proper resources, defining test levels and

cycles, and planning incident management, .

initiate the testing effort and control the execution of te~f setup configuration management of testware for traceabihtv, generate test progress metrics,

decide automation details - what needs to be automated, to what degree, and how,

select tools to support testing and organize appropriate tool usage training for testers,

decide on test environment implementation, schedule tests, and

write test summary reports.





















Given the list above, items ii and iv are not tasks of a test leader; hence, we can eliminate response option B, C, and D.

Answer A is correct.

Reference

ISTQB (2007a), pp. 45-46.

185

Sample Exam QUestions: ISTQB Certified Tester Foundation Level

31. What is a group of test activities that are organized and managed together? (Kl)

A. Test procedure specification

B. Test level

C. Test case specification

D. Test plan ,0

Solution

AU but one of the response options listed above are documents that describe part of or the entire testing process. Test procedure specification enumerates the steps needed to run a test. Test case specification defines the input values, expected results, and executions conditions of a test item. Test plan describes the scope, approach, test items, pass/fail criteria, test resources, and schedule of testing activities. Test level is not a document but a set of test activities or events that are organized and managed collectively. Examples of test levels are component test, integration test, system test, and acceptance test.

Answer B is correct.

References

ISTQB (2007a), p. 47 and ISTQB (2007b), p. 32.

186

Sample Exam Questions: ISTQB Certified Tester Foundation Level

32. Which test approaches or strategies are characterized by the descriptions below? (K2)

5. Process-compliant approaches

T. Heuristic approaches

U. Consultative approaches

V. Regression-averse approaches

1. Includes reuse of existing test material

2. Listens to suggestions from technology experts

3. Adheres to industry-specific standards

4. Runs test execution and evaluation concurrently

A. S4,T3,U2,Vl

B. 51, T2, U3, V4

C. 52, T3, U1, V4

D. S3, T4, U2, Vi

Solution

Approach Characteristic
Process-comQ!iant conforms to indu~-specific standards
simultaneously run test execution and
Heuristic evaluation _iLe. explorato_!Y_ testin_g)
Consultative steered by advice and guidance of
technology and/or business experts
Reqression-averse reuse of existing test material (i.e.,
r~ression testingl Answer D is correct.

Reference

ISTQB (2007a), p. 48.

187

Sample Exam Questions; ISTQB Certified Tester Foundation Level

Sample Exam Questions: ISTQB Certified Tester Foundation Level

34. What are the main objectives of software project risk management? (K2)

33. What is the ratio of the number of failures relative to a category and a unit of measure? (K3)

A. Failure rate A.
B. Defect density
C. Failure mode B.
D. Fault tolerance
C.
Solution
D. Increase focus on preventive processes and improve tester job satisfaction

Reduce the probability of occurrence and decrease

the potential impact

Control contractor problems and minimize the impact of

corporate politics

Increase the probability of project success regardless of the

cost involved

Defect density is the ratio of the number of defects in a component relative to the component's size (measured in total lines of code).

Solution

Failure mode is the physical or functional sign or symptom of a failure (e.g., slow operation, abrupt termination of execution).

Watch out for the word "and" in non-Kl type of questions (response options A, B, and C). There is a very good c~ance that one part of the statement is correct and the other one IS not.

Software project risk analysis entails assessing two key factors: the likelihood of occurrence (probability) and the potential effects (impact) of an event, hazard, threat, or situation. The probability of occurrence and the potential impact must be minimized as part of software project risk management.

Failure rate is the ratio of the number of failures of a given . category to a given unit of measurement (e.g., unit of time, unit of transaction).

Fault tolerance is the ability of the system to maintain a specified level of performance or respond adequately in the event of a failure.

Answer B is correct.

Answer A is correct.

Reference

References

ISTQB (2007a), p. 52.

ISTQB (2007a), p. 49 and ISTQB (2007b), pp. 14, 16, and 17.

188

189

Sample Exam Questions: ISTQB Certified Tester Foundation Level

35. Based on the IEEE Standard for Software Test Documentation (IEEE Std 829-1998), in which sections of the test incident report should the following details be recorded? (K1)

a) Test incident report identifier

b) Summary

c) Incident description

d) Impact

1. Expected results

2. Actual results

3. Procedure step 4, Environment

5. Revision level

6. Date and time

A. a: 3; b: 5; c: 1,2,4, and 6

B. b: 5; c: 1, 2, 3, 4, and 6

C. b: 5 and 6; c: 1, 2, 3, and 4

D. a: 5; c: 1, 2, 3, 4, and 6

Solution

The test incident report identifier (section a) contains only the unique number that identifies the incident report,

The summary (section b) identifies the test items involved indicating the version or revision level (item 5) and reference to appropriate test documents.

The incident description section of the report includes inputs, expected results (item 1), actual results (item 2), anomalies, date, and time (item 6), procedure step (item 3), environment (item 4), repeat attempts, testers, and observers.

The impact section of the test incident report should indicate, if known, the effect the incident will have on test plans or any test specifications (deslqn, procedure, or case).

Answer B is correct.

190

Sample Exa~ Questions: ISTQB Certified Tester Foundation Level

Reference

ISTQB (2007a), p. 54.

191

Sample Exam Questions: ISTQB Certified Tester Foundation Level

Sample Exam Questions: ISTQB Certified Tester Foundation Level

37. Which tool can be used to support and control part of the test management process? (K1)

36. Based on the IEEE Standard for SOftware Test Documentation (IEEE Std 829-1998), which of the following sections are part of the test summary report? (K3)

a) Summary

b) Test incident report identifier

c) Test deliverables

d) Risks and contlnqencies

e) Variances f) .. Approvals

g) Output specifications

A. Coverage management tool

B. Test management tool

C. Data preparation tool

D. Petformance testing tool

Solution

The question itself hints at the correct answer, which clearly is response option B. Response options A, C, and D are tools used to support test execution, test specification, and petformance and monitoring, respectively. Test management tools can be applied to all test activities and encompass the entire system development life cycle.

A. a, e, and f

B. a, c, and d

C. a, b, and f

D. a, d, and e

Solution

Answer B is correct.

A review of the sections listed above shows that section b (test incident report identifier) obviously is not part of a test summary report. Thus, response option C can be eliminated. Sections c (test deliverables) and d (risks and contingencies) are parts more appropriate for test plans rather than the test summary report. Given this, we can further eliminate response options B and Dr hence making response option A the correct answer,

Reference

ISTQB (2007a), p. 57.

Answer A is correct.

Reference

ISTQB (2007a), p. 54.

192

193

Sample Exam Questions: ISTQB Certified Tester Foundation level

Sample Exam Questions: ISTQB certified Tester Foundation Level

39. Which tool needs to interface with other office automation software in order to generate reports in the format required by the organization? (Kl)

38. What is a scripting technique that uses data files to contain not only test data and expected results, but also keywords related to the application being tested? (K1)

A. Automation technique

B. Scripting language

C. Process-driven testing

D. Keyword-driven testing

SOlution

A. Progress tracking tools

8. Test management tools

C. Metrics management tools

D. Test execution tools

Solution

Scripting language is not a technique but rather a programming language with which test scripts are written. Therefore, response opt~on B can be ruled out as the correct answer. Response options A and C, though plausible, were never mentioned nor discussed in the syllabus, thus making both response options flawed.

Progress tracking, metrics analysis and management, and test execution tools all interrelate with test management tools. Although aU the response options may intert:ace with other. automation software to generate reports, it IS response option B that is the superior answer because it encompasses the other test tools.

Answer D is correct.

Answer B is correct.

Reference

Reference

ISTQB (2007a), p. 63.

ISTQB (2007a), pp. 57-63.

194

195

Sample Exam Questions: ISTQB Certified Tester Foundation level

40. Which of the following can be considered as success factors when deploying a new tool in an organization? (K2)

References

A. Providing coaching to users and defining usage guidelines

B. Monitoring tool usage and reducing the need for risk analysis

C. Improving processes and focu~ng more on component testing

D. Assessing testing completion and minimizing code reviews

The Institute of Electrical and Electronics Engineers, Inc. (1997).

IEEE standard fbr software reviews [IEEE Std 1028- 1997]. New York: Author.

The Institute of Electrical and Electronics Engineers, Inc. (1998).

IEEE standard for software test documentation [IEEE Std 829-1998]. New York: Author.

The Institute of Electrical and Electronics Engineers, Inc. (2002).

IEEE standard glossary of software engineering terminology [IEEE Std 610.12-1990 (R2002)]. New York:

Author.

International Software Testing Qualifications Board. (2007a).

Certified tester foundation level syllabus. Erlangen, Germany: Author.

International Software Testing Qualifications Board. (2005).

ISTQB foundation exam format and question writing guidelines. Erlangen, Germany: Author.

International Software Testing Qualifications Board. (2007b).

Standard glossary of terms used in software testing. Erlangen, Germany: Author.

solution

The success factors for tool deployment within an organization are as follows:

• incremental roll-out of the tool to the rest of the

organization,

• adapt and improve processes to fit with tool usage, .. provide training for new users,

• define usage guidelines,

• implement an approach to learn lessons from tool use, and

e monitor tool benefits and usage.

Answer A is correct.

Reference

ISTQB (2007a), p. 64.

196

197

Вам также может понравиться